TEST - 20 (REVISION CUM Total Marks 200 MOCK TEST - 5) :

1 An indicator species is the one whose status provides information on the overall condition of the ecosystem and of other species in that ecosystem. Consider the following species and the indications they show in an ecosystem. 1. Mosses: A. Help indicate acidic soil 2. Lichens: B. Help indicate air pollution 3. Fungi: C. Help indicate old-growth forests where an abundance of coarse woody debris exists

Select the correct match using the codes given below. A. 1A, 2B, 3C B. 1C, 2B, 3A C. 1B, 2C, 3A D. 1A, 2C, 3B

Correct Answer : A

Answer Justification :

Learning: Some of the examples of indicator species are given below.

They reflect the quality and changes in environmental conditions as well as aspects of community composition.

Stoneflies: indicate high oxygen water - Stoneflies spend the majority of their lives as nymphs. Many species require a high concentration of dissolved oxygen and are found in clean swift streams with gravel or stone bottom.

Mosses: some moss species indicate acidic soil. Delicate mosses found on rocks and trees in cities around the world can be used to measure the impact of atmospheric change and could prove a low- cost way to monitor urban pollution.

The “bioindicator” responds to pollution or drought-stress by changing shape, density or disappearing, allowing scientists to calculate atmospheric alterations

NAVEENLichens: some KUMAR species indicate N S - [email protected] air pollution. Lichens as a group have - 9632470826a worldwide distribution and grow almost on any surface, for example soil, bark, roof tiles or stone. Because lichens get all their nutrients from the air, many species are very sensitive to air pollution.

Fungi: Can indicate old-growth forests where an abundance of coarse woody debris exists.

Mollusca: numerous bivalve molluscs indicate water pollution status. Mollusca, and quite often bivalve molluscs are used as bioindicators to monitor the health of an aquatic environment, either fresh- or seawater.

Q Source: Additional Research: http://www.thehindu.com/sci-tech/energy-and-environment/moss-serves-as-a-cheap-pollution-monito r/article19547104.ece

offline.insightsias.com 1 © Insights Active Learning | All rights reserved - 554. You may not reproduce, distribute or exploit the contents in any form without written permission by copyright owner. Copyright infringers may face civil and criminal liability TEST - 20 (REVISION CUM Total Marks 200 MOCK TEST - 5) :

2 Profits or gains arising from transfer of a capital asset are called “Capital Gains” and are charged to tax under the head “Capital Gains”. A gain in the price of which of the following would be considered as Capital gains? 1. Archaeological collections 2. Jewellery 3. A work of art 4. Stocks

Select the correct answer using the codes below. A. 1 and 2 only B. 4 only C. 2 and 4 only D. 1, 2, 3 and 4

Correct Answer : D

Answer Justification :

Justification: Capital asset is defined to include: Any kind of property held by an assessee, whether or not connected with business or profession of the assesse, stocks, movable property, jewellery; archaeological collections; drawings; paintings; sculptures; or any work of art.

For e.g. if you purchased a residential house in 2015 for Rs. 10,00,000 and sold it in 2016 for Rs. 20,00,000, a capital asset gain of Rs. 10,00,000 arising on account of sale of residential house will be charged to tax under the head “Capital Gains”.

Q Source: http://www.incometaxindia.gov.in/Tutorials/15-%20LTCG.pdf

Chapter 17: Indian Economy: Ramesh Singh

NAVEEN KUMAR N S - [email protected] - 9632470826

offline.insightsias.com 2 © Insights Active Learning | All rights reserved - 554. You may not reproduce, distribute or exploit the contents in any form without written permission by copyright owner. Copyright infringers may face civil and criminal liability TEST - 20 (REVISION CUM Total Marks 200 MOCK TEST - 5) :

3 Consider the following statements about Madam Bhikaji Cama. 1. She co-founded the Paris Indian Society. 2. She has served as the president of Indian National Congress (INC). 3. She unfurled the National Flag at the International Socialist Conference in Stuttgart in 1907. 4. She founded the Indian Home Rule Society.

Select the correct answer using the codes below. A. 1 and 3 only B. 2, 3 and 4 only C. 3 only D. 1, 2 and 4 only

Correct Answer : A

Answer Justification :

Justification: Statement 1: She co-founded the Paris Indian Society together with Singh Rewabhai Rana and Munchershah Burjorji Godre.

Statement 2 and 4: She served as the private secretary of Dadabhai Naoroji, the president of the British Committee of the Indian National Congress. Together with Naoroji and Singh Rewabhai Rana, Cama supported the founding of Varma's Indian Home Rule Society in 1905.

Statement 3: In her appeal for human rights, equality and for autonomy from Great Britain, she unfurled what she called the "Flag of Indian Independence"

Q Source: Additional Research: 12th NCERT: Our-pasts III

NAVEEN KUMAR N S - [email protected] - 9632470826

offline.insightsias.com 3 © Insights Active Learning | All rights reserved - 554. You may not reproduce, distribute or exploit the contents in any form without written permission by copyright owner. Copyright infringers may face civil and criminal liability TEST - 20 (REVISION CUM Total Marks 200 MOCK TEST - 5) :

4 Sustainable Development Knowledge Platform (SDKP) is an initiative of

A. United Nations B. International Union for Conservation of Nature (IUCN) C. WWF (World Wildlife Fund) D. Conservation International

Correct Answer : A

Answer Justification :

Learning: The UN Department of Economic and Social Affairs (UNDESA) launched the UN web- based knowledge platform for sustainable development (Sustainable Development Knowledge Platform SDKP).

The site describes different sustainable development activities within the UN system, and it provides resources and links to nearly 6,000 related documents.

The platform presents analysis on the key topics including in the Rio+20 outcome document and contains additional links related to Agenda 21 and the Commission for Sustainable Development.

You can go through it at the Q Source.

Q Source: https://sustainabledevelopment.un.org/

NAVEEN KUMAR N S - [email protected] - 9632470826

offline.insightsias.com 4 © Insights Active Learning | All rights reserved - 554. You may not reproduce, distribute or exploit the contents in any form without written permission by copyright owner. Copyright infringers may face civil and criminal liability TEST - 20 (REVISION CUM Total Marks 200 MOCK TEST - 5) :

5 Consider the following about Very High Resolution (VHR) Satellite Imagery. 1. It offers sub-centimetre resolution. 2. Any type of vehicles on road can be identified using VHR imagery. 3. Structural changes of landscapes can be easily monitored with this imagery.

Select the correct answer using the codes below. A. 1 and 2 only B. 2 and 3 only C. 1 and 3 only D. 1, 2 and 3

Correct Answer : B

Answer Justification :

Justification: It offers sub-meter resolution – one of the highest image qualities currently available from commercial remote sensing satellites.

Sub-meter (VHR) resolution satellites offer users a smaller ground sampling distance (GSD) than other imaging satellites, making them more suitable for reliable natural or man-made, objects and landscapes observation, site monitoring, object identification and many other tasks requiring precision data.

Among its many advantages, VHR images enable the identification of specific models of vehicles, vessels and airplanes, as well as the monitoring of structural changes, natural disasters, industrial operations and livestock. This versatility makes EROS imagery a powerful resource for both civilian and military needs.

This versatility makes VHR imagery a powerful resource to Government officials, which can be integrated for variety of activities. NAVEEN KUMAR N S - [email protected] - 9632470826 Recently Forest Survey of India (FSI) acquired VHR data from SkyMap Global through NRSC for forest areas of Karnataka and Maharashtra.

An application has developed which allow users to visualize VHR images and other datasets.

offline.insightsias.com 5 © Insights Active Learning | All rights reserved - 554. You may not reproduce, distribute or exploit the contents in any form without written permission by copyright owner. Copyright infringers may face civil and criminal liability TEST - 20 (REVISION CUM Total Marks 200 MOCK TEST - 5) :

Q Source: http://117.239.115.44:81/vhr/

NAVEEN KUMAR N S - [email protected] - 9632470826

offline.insightsias.com 6 © Insights Active Learning | All rights reserved - 554. You may not reproduce, distribute or exploit the contents in any form without written permission by copyright owner. Copyright infringers may face civil and criminal liability TEST - 20 (REVISION CUM Total Marks 200 MOCK TEST - 5) :

6 Consider the following statements. A Person of Indian Origin (PIO) card holder 1. doesn’t need a visa to visit India. 2. doesn’t require a student or employment visa to acquire employment or academic opportunities in India. 3. has voting rights equivalent to NRIs

Select the correct answer using the codes below. A. 1 and 2 only B. 2 and 3 only C. 1 and 3 only D. 1, 2 and 3

Correct Answer : A

Answer Justification :

Justification: Simultaneous existence of PIO and OCI cards led to confusion among People of Indian Origin residing abroad.

So, PIO and OCI car were merged which lead to simplification of the rules under a single umbrella.

This is aimed at simplifying the visa-free entry for people of Indian origin into India.

Benefits of a PIO card:

A PIO card holder doesn’t need a visa to visit India. The holder also doesn’t require a student or employment visa to acquire employment or academic opportunities in India.

The holder is also exempted from registering at the foreigner regional registration office (FRRO) during the duration of stay in India. NAVEEN KUMAR N S - [email protected] - 9632470826 The holder also enjoys parity with NRIs in concern to economic, financial and educational matters. These may include matters related to property transfer or acquisition, holding, disposal, investment, admission of children in educational institutions under general category quota for NRIs.

Separate immigration counters are provided at all International airports in India for PIO card holders.

However, it does not provide voting rights to the holder. Prior permission is needed to undertake mountaineering expeditions or any such related research work in protected areas.

Q Source: http://www.insightsonindia.com/2018/01/01/insights-daily-current-affairs-01-january-2018/ offline.insightsias.com 7 © Insights Active Learning | All rights reserved - 554. You may not reproduce, distribute or exploit the contents in any form without written permission by copyright owner. Copyright infringers may face civil and criminal liability TEST - 20 (REVISION CUM Total Marks 200 MOCK TEST - 5) :

7 Natura 2000 is

A. the largest coordinated network of protected areas in the world B. a GHG emission standard of European Union (EU) recently admitted by the Board of Emissions in UNEP C. a list of most vulnerable species as compiled by the IUCN D. a non-profit organization that helps conserve small islands in LDCs

Correct Answer : A

Answer Justification :

Learning: Natura 2000 is a network of core breeding and resting sites for rare and threatened species, and some rare natural habitat types which are protected in their own right.

It stretches across all 28 EU countries, both on land and at sea.

The aim of the network is to ensure the long-term survival of Europe's most valuable and threatened species and habitats, listed under both the Birds Directive and the Habitats Directive.

Natura 2000 is not a system of strict nature reserves from which all human activities would be excluded. While it includes strictly protected nature reserves, most of the land remains privately owned.

The approach to conservation and sustainable use of the Natura 2000 areas is much wider, largely centered on people working with nature rather than against it.

Q Source: http://ec.europa.eu/environment/nature/natura2000/index_en.htm

NAVEEN KUMAR N S - [email protected] - 9632470826

offline.insightsias.com 8 © Insights Active Learning | All rights reserved - 554. You may not reproduce, distribute or exploit the contents in any form without written permission by copyright owner. Copyright infringers may face civil and criminal liability TEST - 20 (REVISION CUM Total Marks 200 MOCK TEST - 5) :

8 What do you understand by ‘responsible government’ that the British tried to incorporate into colonial India by several enactments? 1. That the government is now responsible for the social security and welfare of people at large 2. That the government would follow the Rule of Law 3. That the Bureaucracy will be nominated by the representatives of the people 4. That the Governor was required to act with the advice of ministers responsible to the provincial legislature.

Select the correct answer using the codes below. A. 1, 2 and 4 only B. 1 and 4 only C. 4 only D. 2 and 3 only

Correct Answer : C

Answer Justification :

Justification: Responsible government is a system which embodies the principle of parliamentary accountability, such as in India.

It is also the foundation of the Westminster system of parliamentary democracy.

Government is responsible to the parliament rather than to the monarch, or, in a colonial context, to the imperial government.

If the parliament is bicameral, then the government is responsible first to the parliament's lower house, which is more numerous, directly elected and thus more representative than the upper house.

Several enactments gradually increased the responsibility of the Ministers and officials to the legislature and indirectly to the people, for e.g. GoI Act, 1935 created a dual system where some subjects were to be administered by the Governor based on Ministerial advice. NAVEEN KUMAR N S - [email protected] - 9632470826 Q Source: Chapter 1: Indian Polity: M Laxmikanth

offline.insightsias.com 9 © Insights Active Learning | All rights reserved - 554. You may not reproduce, distribute or exploit the contents in any form without written permission by copyright owner. Copyright infringers may face civil and criminal liability TEST - 20 (REVISION CUM Total Marks 200 MOCK TEST - 5) :

9 Aerogels find potential application in 1. Improving drug delivery 2. Oil spill cleaning 3. Heat insulation materials for buildings

Select the correct answer using the codes below. A. 1 and 2 only B. 2 and 3 only C. 3 only D. 1, 2 and 3

Correct Answer : D

Answer Justification :

Justification: Scientists recently successfully converted paper waste into green nontoxic cellulose aerogels.

Statement 1: It can be used in a drug delivery system owing to its biocompatibility. Due to its high surface area and porous structure, drugs can be adsorbed from supercritical CO2. The release rate of the drugs can be tailored by varying the properties of the aerogel.

Statement 2: It can be used as a chemical adsorber for cleaning up spills.

Statement 3: Aerogel is a synthetic porous ultralight material derived from a gel, in which the liquid component of the gel has been replaced with a gas. The result is a solid with extremely low density and low thermal conductivity. It is nearly 98% air. So, it fits as a heat insulation material.

Q Source: https://www.businesswire.com/news/home/20180110005618/en/Global-Aerogel-Market-Analysis-201 7-Forecasts-2018-2022

NAVEEN KUMAR N S - [email protected] - 9632470826

offline.insightsias.com 10 © Insights Active Learning | All rights reserved - 554. You may not reproduce, distribute or exploit the contents in any form without written permission by copyright owner. Copyright infringers may face civil and criminal liability TEST - 20 (REVISION CUM Total Marks 200 MOCK TEST - 5) :

10 Earth's atmosphere is composed primarily of nitrogen and oxygen. Consider the following about these gases. 1. These gases are transparent to outgoing solar radiation. 2. They do not absorb or emit solar or infrared radiation.

Which of the above is/are correct? A. 1 only B. 2 only C. Both 1 and 2 D. None

Correct Answer : C

Answer Justification :

Justification: They are transparent to outgoing infrared radiation, which means that they do not absorb or emit solar or infrared radiation. The image below explains the absorption by different gases.

However, there are other gases in Earth's atmosphere that do absorb infrared radiation. These gases are known as greenhouse gases.

Water vapor (H2O) is the strongest greenhouse gas, and the concentration of this gas is largely controlled by the temperature of the atmosphere. As air becomes warmer, it can hold more moisture or water vapor.

NAVEEN KUMAR N S - [email protected] - 9632470826

offline.insightsias.com 11 © Insights Active Learning | All rights reserved - 554. You may not reproduce, distribute or exploit the contents in any form without written permission by copyright owner. Copyright infringers may face civil and criminal liability TEST - 20 (REVISION CUM Total Marks 200 MOCK TEST - 5) :

Q Source: http://www.ces.fau.edu/nasa/module-2/how-greenhouse-effect-works.php NAVEEN KUMAR N S - [email protected] - 9632470826

offline.insightsias.com 12 © Insights Active Learning | All rights reserved - 554. You may not reproduce, distribute or exploit the contents in any form without written permission by copyright owner. Copyright infringers may face civil and criminal liability TEST - 20 (REVISION CUM Total Marks 200 MOCK TEST - 5) :

11 With reference to the Trade Promotion Council of India, consider the following statements. 1. It is recognized and supported by the Department of Commerce. 2. It supports India’s trade policy and economic diplomacy through research. 3. Members of TPCI get automated funding for business expansion in India and abroad.

Select the correct answer using the codes below. A. 1 and 2 only B. 2 and 3 only C. 1 and 3 only D. 1, 2 and 3

Correct Answer : A

Answer Justification :

Justification: Statement 1 and 2: TPCI is an apex trade and investment promotion organization notified in the Foreign Trade Policy. TPCI is also recognized and supported by the Department of Commerce, Govt. of India. We work towards facilitating the growth of Indian industry with global investment & trade opportunities.

The council provides strategies for expanding business internationally, by organizing specialized business events and simultaneously working with the Government by providing policy suggestions which are essentially based on inputs collated from research and industry stakeholders.

Statement 3: Services to Ordinary Members Include

Close interaction with TPCI’s members, international agencies, diplomatic missions in India and Indian missions abroad, academicians and technical experts.

Participation in various trade specific and general business events and gain professional knowledge through seminars, workshops and round table conferences. NAVEEN KUMAR N S - [email protected] - 9632470826 Services to Patron members (apart from the above) include:

Nomination of members on Committees & Advisory Forums constituted by the government of different countries.

Priority consideration for participation in high level delegations visiting India and those visiting other countries and meeting senior government officials.

Patron members are eligible to avail TPCI Business Services on demand.

Q Source:

http://www.tpci.in/about-us/about-tpci/ offline.insightsias.com 13 © Insights Active Learning | All rights reserved - 554. You may not reproduce, distribute or exploit the contents in any form without written permission by copyright owner. Copyright infringers may face civil and criminal liability TEST - 20 (REVISION CUM Total Marks 200 MOCK TEST - 5) :

Additional Research: http://www.insightsonindia.com/2018/01/06/insights-daily-current-affairs-06-january-2018/

NAVEEN KUMAR N S - [email protected] - 9632470826

offline.insightsias.com 14 © Insights Active Learning | All rights reserved - 554. You may not reproduce, distribute or exploit the contents in any form without written permission by copyright owner. Copyright infringers may face civil and criminal liability TEST - 20 (REVISION CUM Total Marks 200 MOCK TEST - 5) :

12 With reference to Chaitanya, consider the following statements. 1. He popularised the Shakti cult in Medieval India. 2. He was against renunciation and asceticism as a path of spiritual liberation. 3. He believed that through song and dance done with love and devotion, one can feel the presence of God.

Select the correct answer using the codes below. A. 1 and 2 only B. 3 only C. 1 only D. 2 and 3 only

Correct Answer : B

Answer Justification :

Justification: He was a well-known saint and reformer of Bengal who popularised the Krishna cult. He renounced the material world, became an ascetic and wandered all over the country preaching his ideas.

He proclaimed the universal brotherhood of man and condemned all distinction based on religion and caste. He emphasised love and peace and showed great sympathy to the sufferings of other people, especially that of the poor and the weak.

He believed that through love and devotion, song and dance, a devotee can feel the presence of God. He accepted disciples from all classes and castes and his teachings are widely followed in Bengal even today.

Q Source: Revision: 7th NCERT: Our pasts –II

NAVEEN KUMAR N S - [email protected] - 9632470826

offline.insightsias.com 15 © Insights Active Learning | All rights reserved - 554. You may not reproduce, distribute or exploit the contents in any form without written permission by copyright owner. Copyright infringers may face civil and criminal liability TEST - 20 (REVISION CUM Total Marks 200 MOCK TEST - 5) :

13 Consider the following areas in India and the rainfall associated with them. 1. Western coast close to the Western Ghats: A. Low Rainfall 2. Western Uttar Pradesh B. High rainfall 3. Northeastern Peninsula - Orissa and Jharkhand: C. Medium rainfall

Select the correct answer using the codes below. A. 1A, 2B, 3C B. 1B, 2A, 3C C. 1C, 2A, 3B D. 1B, 2C, 3A

Correct Answer : B

Answer Justification :

Justification & Learning: Areas of High Rainfall : The highest rainfall occurs along the west coast, on the Western Ghats, as well as in the sub-Himalayan areas is the northeast and the hills of Meghalaya. Here the rainfall exceeds 200 cm. In some parts of Khasi and Jaintia hills, the rainfall exceeds 1,000 cm. In the Brahmaputra valley and the adjoining hills, the rainfall is less then 200 cm.

Areas of Medium Rainfall: Rainfall between 100-200 cm is received in the southern parts of Gujarat, east Tamil Nadu, northeastern Peninsula covering Orissa, Jharkhand, Bihar, eastern Madhya Pradesh, northern Ganga plain along the sub-Himalayas and the Cachar Valley and Manipur.

Areas of Low Rainfall: Western Uttar Pradesh, Delhi, Haryana, Punjab, Jammu and Kashmir, eastern Rajasthan, Gujarat and Deccan Plateau receive rainfall between 50-100 cm.

Areas of Inadequate Rainfall: Parts of the Peninsula, especially in Andhra Pradesh, Karnataka and Maharashtra, Ladakh and most of western Rajasthan receive rainfall below 50 cm.

Q Source: Revision: 9th NCERT Geography

NAVEEN KUMAR N S - [email protected] - 9632470826

offline.insightsias.com 16 © Insights Active Learning | All rights reserved - 554. You may not reproduce, distribute or exploit the contents in any form without written permission by copyright owner. Copyright infringers may face civil and criminal liability TEST - 20 (REVISION CUM Total Marks 200 MOCK TEST - 5) :

14 Countries of Particular Concern (CPC) and Priority Watch List are often seen in news with reference to 1. Indicators of Religious freedom 2. Intellectual property rights deficiencies 3. Openness of the Economy 4. Zones prone to civil wars

Select the correct answer using the codes below. A. 1 and 2 only B. 2 and 3 only C. 1 and 4 only D. 3 and 4 only

Correct Answer : A

Answer Justification :

Justification: CPC: The US State Department has unveiled its list of countries designated as the worst offenders against religious liberty. These countries are labelled as CPC.

A country is labelled as a CPC, by USA, after it engages in “systemic, ongoing, [and] egregious” violations of religious liberty. This declaration is in accordance with the International Religious Freedom Act of 1998.

The list includes Burma, China, Eritrea, Iran, North Korea, Sudan, Saudi Arabia, Tajikistan, Turkmenistan, and Uzbekistan.

The governments that have been designated as CPCs are subject to ‘Presidential Actions’, such as sanctions, which may include economic or political measures directed against a government to encourage it to improve the state of religious freedom in its country. NAVEEN KUMAR N S - [email protected] - 9632470826

offline.insightsias.com 17 © Insights Active Learning | All rights reserved - 554. You may not reproduce, distribute or exploit the contents in any form without written permission by copyright owner. Copyright infringers may face civil and criminal liability TEST - 20 (REVISION CUM Total Marks 200 MOCK TEST - 5) :

Priority Watch List: The Special 301 Report is prepared annually by the Office of the United States Trade Representative (USTR)

The reports identify trade barriers to U.S. companies and products due to the intellectual property laws, such as copyright, patents and trademarks, in other countries

These are some of the designations given to countries:

NAVEENPFC: PriorityKUMAR Foreign N SCountry - [email protected] - 9632470826

PWL: Priority Watch List (India was given this designation)

WL: Watch list

Q Source: http://www.insightsonindia.com/2018/01/05/insights-daily-current-affairs-05-january-2018/

offline.insightsias.com 18 © Insights Active Learning | All rights reserved - 554. You may not reproduce, distribute or exploit the contents in any form without written permission by copyright owner. Copyright infringers may face civil and criminal liability TEST - 20 (REVISION CUM Total Marks 200 MOCK TEST - 5) :

15 MoSAic Observatory is concerned with an improved understanding of the region of

A. Equatorial Rainforests B. Lower Earth orbits C. Arctic Region D. Tropical cyclones

Correct Answer : C

Answer Justification :

Background and Learning: The Arctic is a key area of global climate change, with warming rates exceeding twice the global average.

Many processes in the Arctic climate system are poorly represented in climate models because they are not sufficiently understood.

Understanding of Arctic climate processes is limited by a lack of year round observations in the central Arctic.

The Multidisciplinary drifting Observatory for the study of Arctic climate (MOSAiC) will be the first year-round expedition into the central Arctic exploring the Arctic climate system.

The project with a total budget exceeding 60 Million € has been designed by an international consortium of leading polar research institutions under the umbrella of the International Arctic Science Committee (IASC).

The results of MOSAiC will contribute to enhance understanding of the regional and global consequences of Arctic climate change and sea-ice loss and improve weather and climate NAVEENpredictions. KUMAR N S - [email protected] - 9632470826

As such it will support safer maritime and offshore operations, contribute to an improved scientific basis for future fishery and traffic along northern sea routes, increase coastal- community resilience, and support science-informed decision-making and policy development.

Q Source: http://www.mosaicobservatory.org/

offline.insightsias.com 19 © Insights Active Learning | All rights reserved - 554. You may not reproduce, distribute or exploit the contents in any form without written permission by copyright owner. Copyright infringers may face civil and criminal liability TEST - 20 (REVISION CUM Total Marks 200 MOCK TEST - 5) :

16 Consider the following UN bodies and programmes which are actively operating in India and their concerned nodal ministries in India. 1. UNDP: Department of Economic Affairs 2. International Labour Organization (ILO): Ministry of Labour and Employment 3. UN Women: Ministry of Women and Child Development 4. UNICEF: Ministry of Social Justice and Empowerment

Select the correct answer using the codes below. A. 1, 2 and 3 only B. 2 and 4 only C. 1 and 4 only D. 1, 2, 3 and 4

Correct Answer : A

Answer Justification :

Justification: Statement 1: Focus Areas: Democratic Governance; Poverty Reduction; Environment and Energy

Nodal Ministry: Department of Economic Affairs, Ministry of Finance

Flagship Publication/s: Global Human Development Report

Statement 2: ILO: Focus Areas: Child Labour, Employment Promotion, Equality and Discrimination, Green jobs, Informal Economy, International Labour Standards, Labour migration, Safety and health at work, Skills and employability, Social security, Workers’ and Employers’ Organizations

Nodal Ministry: Ministry of Labour and Employment

Flagship Publication/s: World of Work Reports

Statement 3: Focus Areas: Ending Violence against Women; Promoting Women’s Leadership and NAVEENParticipation; KUMAR National PlanningN S - [email protected] Budgeting; Women’s Economic Empowerment; - 9632470826 Peace and Security.

Nodal Ministries: Ministry of Women and Child Development

Flagship Publication/s: Progress of the World’s Women

Statement 4: Focus Areas: Child Development and Nutrition; Child Protection; Education, Child Environment; Polio Eradication; Reproductive and Child Health; Children and AIDS; Social Policy, Planning, Monitoring and Evaluation; Advocacy and Partnership; Behaviour Change Communication; Emergency Preparedness and Response.

Nodal Ministries: Ministry of Women and Child Development

Flagship Publication/s: The State of the World’s Children; Progress for Children

Q Source: http://in.one.un.org/who-we-are/ offline.insightsias.com 20 © Insights Active Learning | All rights reserved - 554. You may not reproduce, distribute or exploit the contents in any form without written permission by copyright owner. Copyright infringers may face civil and criminal liability TEST - 20 (REVISION CUM Total Marks 200 MOCK TEST - 5) :

17 Peninsular rivers do not carry a large quantity of silt as well as are characterised by absence of meanders. This can be due to which of these reasons? 1. These are fast flowing rivers due to which formation of meanders become difficult. 2. There rivers pass through hard rocky terrain due to erosion capacity of the river is not effective.

Which of the above is/are correct? A. 1 only B. 2 only C. Both 1 and 2 D. None

Correct Answer : C

Answer Justification :

Justification: Peninsular rivers flow at fast speeds. When they slow down, lateral bank cutting is predominant.

Any river that flow at a considerable speed can erode both lateral banks as well as river bed.

However, this erosion can take place only when the bank is soft or river bed is made of erodible soft rocks.

Since, peninsular blocks are hard, rocky, old and consolidated, little silt is accumulated by these rivers.

Same is true for meanders, for meanders to form rivers must slow down and the terrain must be soft.

Q Source: Revision: Chapter 3: 11th NCERT: India Physical Geography

NAVEEN KUMAR N S - [email protected] - 9632470826

offline.insightsias.com 21 © Insights Active Learning | All rights reserved - 554. You may not reproduce, distribute or exploit the contents in any form without written permission by copyright owner. Copyright infringers may face civil and criminal liability TEST - 20 (REVISION CUM Total Marks 200 MOCK TEST - 5) :

18 Access and Benefit-sharing (ABS) is a key idea in which of the following environmental conventions?

A. Convention on Biological Diversity B. CITES Convention C. United Nations Framework Convention on Climate Change (UNFCC) D. Washington Convention, 2001

Correct Answer : A

Answer Justification :

Learning: The Nagoya Protocol on access to genetic resources and the fair and equitable sharing of benefits arising from their utilisation is a supplementary agreement to the Convention on Biological Diversity, which provides a transparent legal framework for the effective implementation of one of the three objectives of the CBD: the fair and equitable sharing of benefits arising out of the utilization of genetic resources.

The ABS Clearing-House is a key tool for facilitating the implementation of the Nagoya Protocol, by enhancing legal certainty and transparency on procedures for access and benefit-sharing, and for monitoring the utilization of genetic resources along the value chain, including through the internationally recognized certificate of compliance.

Q Source: http://www.biodiversitya-z.org/content/nagoya-protocol

NAVEEN KUMAR N S - [email protected] - 9632470826

offline.insightsias.com 22 © Insights Active Learning | All rights reserved - 554. You may not reproduce, distribute or exploit the contents in any form without written permission by copyright owner. Copyright infringers may face civil and criminal liability TEST - 20 (REVISION CUM Total Marks 200 MOCK TEST - 5) :

19 Consider the following about Nilambur teaks. 1. They are found in dry deciduous climate zones. 2. The teak is resistance to fungal decay. 3. The tree has anti-oxidant properties.

Select the correct answer using the codes below. A. 1 and 2 only B. 2 and 3 only C. 3 only D. 1, 2 and 3 only

Correct Answer : B

Answer Justification :

Justification: Kerala’s Nilambur teak aka Malabar teak has found its place in the Geographical Indications (GI) Registry.

The golden brown teak is known for its log dimensions, desired wood figure and wide reputation in the world of trade.

Its durability is attributed to the synergistic effect of total extensive components and the resistance to fungal decay to naphthoquinone and derivatives it contains. The hydrophobicity, anti-oxidant properties and oily nature are due to a caoutchouc compound.

The Nilambur forests are also known for their tribal settlements, vast rain forests, waterfalls and the world's first Teak Museum.

Q Source: https://en.wikipedia.org/wiki/Nilambur

http://www.insightsonindia.com/2018/01/03/insights-daily-current-affairs-03-january-2018/

NAVEEN KUMAR N S - [email protected] - 9632470826

offline.insightsias.com 23 © Insights Active Learning | All rights reserved - 554. You may not reproduce, distribute or exploit the contents in any form without written permission by copyright owner. Copyright infringers may face civil and criminal liability TEST - 20 (REVISION CUM Total Marks 200 MOCK TEST - 5) :

20 If Iceland, Finland, Turkey and Spain are the four vortexes of a Quadrilateral, assuming the vortexes to be at the geometrical centre of each nation, the area of the quadrilateral would encompass which of these regions? 1. Caspian Sea 2. Poland 3. Italy 4. North Sea

Select the correct answer using the codes below. A. 1 and 2 only B. 2 and 3 only C. 1 and 4 only D. 2 and 4 only

Correct Answer : B

Answer Justification :

Learning:

NAVEEN KUMAR N S - [email protected] - 9632470826

Q Source: Map-based: Europe

offline.insightsias.com 24 © Insights Active Learning | All rights reserved - 554. You may not reproduce, distribute or exploit the contents in any form without written permission by copyright owner. Copyright infringers may face civil and criminal liability TEST - 20 (REVISION CUM Total Marks 200 MOCK TEST - 5) :

21 With reference to the DAM REHABILITATION AND IMPROVEMENT PROJECT (DRIP), consider the following statements. 1. The Ministry of Water Resources (MoWR), with assistance from the World Bank, is implementing this project. 2. The project will promote new technologies for dam safety evaluation and implementation in some identified premier academic and research institutes of the country.

Which of the above is/are correct? A. 1 only B. 2 only C. Both 1 and 2 D. None

Correct Answer : C

Answer Justification :

Background: Presently, India ranks third globally with 5254 large dams in operation and about 447 are under construction.

In addition, there are several thousand smaller dams. These dams are vital for ensuring the water security of the Country; and these also constitute a major responsibility in terms of asset management and safety.

In 2012, the Central Water Commission (CWC) under Ministry of Water Resources, River Development & Ganga Rejuvenation with assistance from the World Bank, embarked upon the six year Dam Rehabilitation and Improvement Project (DRIP) at an estimated cost of Rs. 2100 crore.

Justification: Statement 1: It would be a six-year project.

The project originally envisaged the rehabilitation and improvement of about 223 dams within four states namely, Kerala, Madhya Pradesh, Odisha, and Tamil Nadu and later Karnataka, Uttarakhand (UNVNL) and Jharkhand (DVC) joined DRIP and total number of dams covered under DRIP NAVEENincreased to KUMAR 250. N S - [email protected] - 9632470826

Statement 2: The project development objectives of DRIP are: (i) to improve the safety and performance of selected existing dams and associated appurtenances in a sustainable manner, and (ii) to strengthen the dam safety institutional setup in participating states as well as at central level.

Q Source: https://www.damsafety.in/

http://www.insightsonindia.com/2018/01/06/insights-daily-current-affairs-06-january-2018/

offline.insightsias.com 25 © Insights Active Learning | All rights reserved - 554. You may not reproduce, distribute or exploit the contents in any form without written permission by copyright owner. Copyright infringers may face civil and criminal liability TEST - 20 (REVISION CUM Total Marks 200 MOCK TEST - 5) :

22 Consider the following statements about the Rig Vedic society. 1. The Rig Vedic society was matriarchal. 2. Women were allowed to participate in literary activities. 3. The basic unit of society was the family whose head was called as Gahapati.

Select the correct answer using the codes below. A. 1 only B. 2 and 3 only C. 1 and 3 only D. 1, 2 and 3

Correct Answer : B

Answer Justification :

Justification: We have had poets like Apala, Viswavara, Ghosa and Lopmudra during the Rig Vedic period.

Women could even attend the popular assemblies. There was no child marriage and the practice of sati was absent.

However, the society was patriarchal. The basic unit of society was family or graham. The head of the family was known as grahapathi.

Monogamy was generally practiced while polygamy was prevalent among the royal and noble families. The wife took care of the household and participated in all the major ceremonies.

Q Source: Revision: Chapter 3: 11th Std. TamilNadu History Textbook

NAVEEN KUMAR N S - [email protected] - 9632470826

offline.insightsias.com 26 © Insights Active Learning | All rights reserved - 554. You may not reproduce, distribute or exploit the contents in any form without written permission by copyright owner. Copyright infringers may face civil and criminal liability TEST - 20 (REVISION CUM Total Marks 200 MOCK TEST - 5) :

23 Red panda is native to

A. Eastern Himalayas and south-western China B. West Asia C. Mediterranean Region D. Siberia and Central Asia

Correct Answer : A

Answer Justification :

Justification: Distribution of the red panda is disjointed, with two extant subspecies:

Western red panda lives in the western part of its range, in Nepal, Assam, Sikkim, and Bhutan.

Styan's red panda lives in the east-north-eastern part of its range, in southern China and northern Burma.

It is also covered under CITES. Habitat loss, poaching and hunting are the biggest threats to red panda.

It is a solitary animal, mainly active from dusk to dawn, and is largely sedentary during the day.

Q Source: Major Species: WWF Website

NAVEEN KUMAR N S - [email protected] - 9632470826

offline.insightsias.com 27 © Insights Active Learning | All rights reserved - 554. You may not reproduce, distribute or exploit the contents in any form without written permission by copyright owner. Copyright infringers may face civil and criminal liability TEST - 20 (REVISION CUM Total Marks 200 MOCK TEST - 5) :

24 What are the possible ways to increase the human capital of the country? 1. Increase the population of the nation 2. Invest in health expenditure

Select the correct answer using the codes below. A. 1 only B. 2 only C. Both 1 and 2 D. None of the above

Correct Answer : C

Answer Justification :

Justification: Statement 1: Human capital can be increased in two important ways. One, that we increase the population and thus increase the labour force and second if the population is fixed or declining, increase expenditure on human resources. Health expenditure directly increases the supply of healthy labour force and is, thus, a source of human capital formation.

Digital literacy builds IT skills, increases productivity and efficiency of labour and digital information is necessary to make decisions regarding investments in human capital as well as for efficient utilisation of the acquired human capital stock.

Q Source: Revision: 11th NCERT: Fundamentals of Human Geography

NAVEEN KUMAR N S - [email protected] - 9632470826

offline.insightsias.com 28 © Insights Active Learning | All rights reserved - 554. You may not reproduce, distribute or exploit the contents in any form without written permission by copyright owner. Copyright infringers may face civil and criminal liability TEST - 20 (REVISION CUM Total Marks 200 MOCK TEST - 5) :

25 With reference to Asian Infrastructure and Investment Bank (AIIB), consider the following statements. 1. It offers sovereign and non-sovereign financing for sustainable projects in energy, telecommunications and rural infrastructure. 2. It may participate in the underwriting of securities issued by any entity or enterprise for purposes consistent with its purpose.

Which of the above is/are correct? A. 1 only B. 2 only C. Both 1 and 2 D. None

Correct Answer : C

Answer Justification :

Justification: According to the Articles of Agreement (AOA) of AIIB, the Bank will “provide or facilitate financing to any member, or any agency, instrumentality or political subdivision thereof, or any entity or enterprise operating in the territory of a member, as well as to international or regional agencies or entities concerned with economic development of the Asia region.”

Furthermore, the AOA permits the Bank to provide financing in a variety of ways, including, inter alia, making loans, investing in the equity capital of an enterprise, and guaranteeing, whether as primary or secondary obligor, in whole or in part, loans for economic development.

In addition, the Bank may underwrite, or participate in the underwriting of, securities issued by any entity or enterprise for purposes consistent with its purpose.

Other areas of Bank finance include agriculture development, water supply and sanitation, environmental protection, and urban development and logistics.

Q Source: Revision: Ramesh Singh: Indian Economy: NAVEENhttps://www.aiib.org/en/about-aiib/who-we-are/financing-operations/index.html KUMAR N S - [email protected] - 9632470826

offline.insightsias.com 29 © Insights Active Learning | All rights reserved - 554. You may not reproduce, distribute or exploit the contents in any form without written permission by copyright owner. Copyright infringers may face civil and criminal liability TEST - 20 (REVISION CUM Total Marks 200 MOCK TEST - 5) :

26 Which one of the following statements about the Treaty of Allahabad is correct?

A. It was concerned with the Battle of Plassey. B. It was imposed by force on the incumbent Mughal Emperor and deprived the Emperor of his property and land in lieu of protection. C. The rights given by this treaty allowed the Company to collect revenue directly from the people certain provinces. D. None of the above is correct.

Correct Answer : C

Answer Justification :

Justification and Learning: It was signed in 1765, between the Mughal Emperor Shah Alam II, and Robert, Lord Clive as a result of the Battle of Buxar of 1764.

The Treaty marks the political and constitutional involvement and the beginning of British rule in India.

Based on the terms of the agreement, Alam granted the East India Company Diwani rights, or the right to collect taxes on behalf of the Emperor from the eastern province of Bengal-Bihar- Orissa.

Thus East India Company got appointed as the imperial tax collector for the Eastern province (Bengal-Bihar-Orissa).

These rights allowed the Company to collect revenue directly from the people of Bengal, Bihar and Orissa. In return, the Company paid an annual tribute of twenty-six lakhs of rupees (equal to 260,000 pounds sterling) while securing for Shah Alam II the districts of Kora and Allahabad. NAVEEN KUMAR N S - [email protected] - 9632470826

The tribute money paid to the emperor was for the maintenance of the Emperor's court in Allahabad.

Q Source: Additional Research: UPSC CAPF 2017

offline.insightsias.com 30 © Insights Active Learning | All rights reserved - 554. You may not reproduce, distribute or exploit the contents in any form without written permission by copyright owner. Copyright infringers may face civil and criminal liability TEST - 20 (REVISION CUM Total Marks 200 MOCK TEST - 5) :

27 There is a growing consciousness of climate change in trade and commerce. In the context, consider the following statements. Assertion (A): There are no WTO rules specific to climate change. Reason (R): Trade is not related to carbon emissions.

In the context of the above, which of these is correct? A. A is correct, and R is an appropriate explanation of A. B. A is correct, but R is not an appropriate explanation of A. C. A is correct, but R is incorrect. D. Both A and R are incorrect.

Correct Answer : C

Answer Justification :

Justification: As per WTO’s website, “The issue of climate change, per se, is not part of the WTO's ongoing work programme and there are no WTO rules specific to climate change. However, the WTO is relevant because climate change measures and policies intersect with international trade in a number of different ways.”

However, the WTO offers a powerful supporting framework for sustainable development and green economy.

It provides an enabling environment through its objectives, institutions and monitoring of potential trade protectionism, enforcement mechanism, toolbox of rules, and growing case law in the environment area.

Q Source: https://www.wto.org/english/tratop_e/envir_e/envir_e.htm

NAVEEN KUMAR N S - [email protected] - 9632470826

offline.insightsias.com 31 © Insights Active Learning | All rights reserved - 554. You may not reproduce, distribute or exploit the contents in any form without written permission by copyright owner. Copyright infringers may face civil and criminal liability TEST - 20 (REVISION CUM Total Marks 200 MOCK TEST - 5) :

28 NABSKILL is a digital interface to promote Skill INDIA. It is an initiative by

A. Reserve Bank of India B. National Skill Development Corporation of India C. National Agricultural Bank for Rural Development D. Labour Bureau

Correct Answer : C

Answer Justification :

Learning: The portal capture the details of all stakeholders (Training Seeker, Training Provider, Placement Agencies and NABARD) involved in skill development initiatives of NABARD.

The eligible training provider can check for their eligibility criteria under “Guidelines” in home page and upon successful registration the training provider can apply online for seeking grant support from NABARD.

The portal captures the details of training related data viz., trainees profiles, training programme details, post training settlement rate etc.

The registered placement agencies can view the details of successful trainees (Name, Type of training obtained, Details of Training Institute) who have undergone skill training supported by NABARD. The placement agencies can approach the training providers for further information on placement, etc.

NAVEEN KUMAR N S - [email protected] - 9632470826

offline.insightsias.com 32 © Insights Active Learning | All rights reserved - 554. You may not reproduce, distribute or exploit the contents in any form without written permission by copyright owner. Copyright infringers may face civil and criminal liability TEST - 20 (REVISION CUM Total Marks 200 MOCK TEST - 5) :

Q Source: http://www.nabskillnabard.org/aboutus.php?language=LG-1&status=Active

NAVEEN KUMAR N S - [email protected] - 9632470826

offline.insightsias.com 33 © Insights Active Learning | All rights reserved - 554. You may not reproduce, distribute or exploit the contents in any form without written permission by copyright owner. Copyright infringers may face civil and criminal liability TEST - 20 (REVISION CUM Total Marks 200 MOCK TEST - 5) :

29 Which of the following statements is correct about Indirect Tax?

A. The incidence and impact of taxation are at different points. B. Only producers pay indirect taxes. C. Interest tax is a form of indirect tax. D. None of the above

Correct Answer : A

Answer Justification :

Justification: The point where tax makes its effect felt is known as the impact of tax—the after effect of tax imposition.

The tax which has incidence and impact at the different points is the indirect tax—the person who is hit does not bleed’ someone else bleeds.

As, for example, excise, sales tax, etc., which are imposed on either the producers or the traders, but it is the general consumers who bear the burden of tax.

The tax which has incidence and impact both at the same point is the direct tax—the person who is hit, the same person bleeds. As for example income tax, interest tax, etc.

Q Source: Chapter 17: Indian Economy: Ramesh Singh

NAVEEN KUMAR N S - [email protected] - 9632470826

offline.insightsias.com 34 © Insights Active Learning | All rights reserved - 554. You may not reproduce, distribute or exploit the contents in any form without written permission by copyright owner. Copyright infringers may face civil and criminal liability TEST - 20 (REVISION CUM Total Marks 200 MOCK TEST - 5) :

30 Arrange these geographical regions from the West to east. 1. Gulf of Aden 2. Laccadive Sea 3. Gulf of Thailand 4. South China Sea

Select the correct answer using the codes below. A. 1234 B. 2143 C. 2134 D. 1243

Correct Answer : A

Answer Justification :

Learning:

NAVEEN KUMAR N S - [email protected] - 9632470826 Q Source: Map-based: South Asia

offline.insightsias.com 35 © Insights Active Learning | All rights reserved - 554. You may not reproduce, distribute or exploit the contents in any form without written permission by copyright owner. Copyright infringers may face civil and criminal liability TEST - 20 (REVISION CUM Total Marks 200 MOCK TEST - 5) :

31 What are the potential impacts of climate change due to extreme weather events and changes in the Oceans? 1. Salinization of irrigation water in the coastal areas 2. Increased coastal erosion 3. Higher frequency of Tsunamis

Select the correct answer using the codes below. A. 1 and 2 only B. 2 and 3 only C. 1 and 3 only D. 1, 2 and 3

Correct Answer : A

Answer Justification :

Justification: Statement 1: Higher sea levels result in higher pore pressure of water due to increased column height. Freshwater is stored beneath the ground, but the sea water will start intruding as the pressure of sea water increase beyond that of the fresh water stored in the ground.

Statement 2: As the ocean level increases, and ocean water intrudes further inland, the coasts will be eroded more and more. Further, the absence of coral reefs and mangroves at a large number of coastal sites makes the process easier.

Statement 3: Tsunamis are tectonic phenomenon, and not related with changes in the ocean levels or atmospheric changes.

The table below encapsulates all major events and their potential impact.

NAVEEN KUMAR N S - [email protected] - 9632470826

offline.insightsias.com 36 © Insights Active Learning | All rights reserved - 554. You may not reproduce, distribute or exploit the contents in any form without written permission by copyright owner. Copyright infringers may face civil and criminal liability TEST - 20 (REVISION CUM Total Marks 200 MOCK TEST - 5) :

NAVEEN KUMAR N S - [email protected] - 9632470826

Q Source: Revision: Topics based on Environmental law/treaties etc. offline.insightsias.com 37 © Insights Active Learning | All rights reserved - 554. You may not reproduce, distribute or exploit the contents in any form without written permission by copyright owner. Copyright infringers may face civil and criminal liability TEST - 20 (REVISION CUM Total Marks 200 MOCK TEST - 5) :

32 External Aids is the best means to finance a government’s fiscal deficit because 1. It brings in foreign currency that is also useful to bridge the Balance of payments (BoP) apart from its utility in developmental expenditures. 2. It does not cause crowding out effect in the domestic market and is favourable to the domestic borrowers.

Which of the above is/are correct? A. 1 only B. 2 only C. Both 1 and 2 D. None

Correct Answer : C

Answer Justification :

Justification: Statement 1: If they are a grant or coming without interest, no better way to finance the deficit, if we ignore their inflationary effects.

Statement 2: When the domestic market has limited amount of funds, and if the government desires to borrow a large share of it to finance the fiscal deficit, it tends to raise the demand for funds in the market. This shoots the market interest rate for the funds and causes problems to the domestic investors who now have to pay a higher interest rate to avail the same loan.

If the same money is borrowed from abroad, the crowing out effect doesn’t occur.

Q Source: Chapter 18: Indian Economy: Ramesh Singh

NAVEEN KUMAR N S - [email protected] - 9632470826

offline.insightsias.com 38 © Insights Active Learning | All rights reserved - 554. You may not reproduce, distribute or exploit the contents in any form without written permission by copyright owner. Copyright infringers may face civil and criminal liability TEST - 20 (REVISION CUM Total Marks 200 MOCK TEST - 5) :

33 Consider the following about the idea of Total Revolution as given by Jayprakash narayan. 1. It demanded annihilation of British rule from India. 2. It desired social transformation on the lines of the ideals of the Sarvodaya.

Which of the above is/are correct? A. 1 only B. 2 only C. Both 1 and 2 D. None

Correct Answer : B

Answer Justification :

Justification: On his return home to Patna in 1975 after release from detention due to acute illness, JP wrote a long letter to his followers explaining the circumstances of the Bihar movement and committing the remaining years of his life to total revolution.

As per him, the objective of this movement was not merely to change the Government, but also to change the society and the individual.

He laid out a plan for the movement; on how the revolution should move on the propaganda for public education, and constructive programmes consisting of creating consciousness about various social evils such as dowry system, caste conflicts, communalism and untouchability.

Thus, JP had a very clear perspective of the social ideals and objectives when he took upon the task of guiding the Indian masses for a concrete action.

JP strongly believed that for democracy to be a lively and effective instrument there is an urgent need for a strong opposition, powerful public opinion, free and fearless press, ideological and moral NAVEENpressure from KUMAR the academicians, N S - [email protected] and trade unions. He, in fact, advised -the 9632470826 people to revise their thinking and attitude towards democratic functioning in India.

Q Source: http://vle.du.ac.in/file.php/617/Jayaprakash_Narayan_on_Democracy/Jayaprakash_Narayan_on_Dem ocracy.pdf

offline.insightsias.com 39 © Insights Active Learning | All rights reserved - 554. You may not reproduce, distribute or exploit the contents in any form without written permission by copyright owner. Copyright infringers may face civil and criminal liability TEST - 20 (REVISION CUM Total Marks 200 MOCK TEST - 5) :

34 Which of the following is INCORRECT regarding revenue expenditure of the Government of India?

A. These expenditures usually do not directly involve creation of capital assets. B. Salaries paid to the Government employees come under revenue expenditure. C. Expenditures on social services are excluded from this category because they augment the human capital of the country. D. Grants given by the government to Indian states and foreign countries come under revenue expenditure.

Correct Answer : C

Answer Justification :

Justification & Learning: The basic identity of such expenditures is that they are of consumptive kind and do not involve creation of productive assets.

They are either used in running of a productive process or running a government.

A broad category of things that fall under such expenditures in India are:

Interest payment by the government on the internal and external loans;

Salaries, Pension and Provident Fund paid by the government to government employees;

Subsidies forwarded to all sectors by the government;

Defence expenditures by the government;

Postal Deficits of the government; NAVEEN KUMAR N S - [email protected] - 9632470826 Law and order expenditures (i.e.. police & paramilitary);

Expenditures on social services (includes all social sector expenditures as education, health care, social security, poverty alleviation, etc.) and general services (tax collection, etc.);

Grants given by the government to Indian states and foreign countries.

Q Source: Chapter 18: Indian Economy: Ramesh Singh

offline.insightsias.com 40 © Insights Active Learning | All rights reserved - 554. You may not reproduce, distribute or exploit the contents in any form without written permission by copyright owner. Copyright infringers may face civil and criminal liability TEST - 20 (REVISION CUM Total Marks 200 MOCK TEST - 5) :

35 What is the role of National Knowledge Network (NKN)? 1. Establishing a high-speed backbone connectivity which will enable knowledge and information sharing amongst NKN connected institutes 2. Developed peer groups of Indian and Foreign Universities for exchange programmes 3. Facilitating advanced distance education in specialized fields like engineering and medicine

Select the correct answer using the codes below. A. 1 and 2 only B. 2 and 3 only C. 1 and 3 only D. 1, 2 and 3

Correct Answer : C

Answer Justification :

Justification: It is a multi-gigabit pan-India network which facilitates the development of India’s communications infrastructure, stimulates research and creates next generation applications and services.

With its multi-gigabit capability, NKN aims to connect all universities, research institutions, libraries, laboratories, healthcare and agricultural institutions across the country to address such paradigm shift.

Mandate of NKN involves:

Enabling collaborative research, development and innovation amongst NKN connected institutes

Facilitating advanced distance education in specialized fields like engineering, science, medicine etc. NAVEEN KUMAR N S - [email protected] - 9632470826 Facilitating an ultra-high speed e-governance backbone

Facilitating connection between different sectoral networks in the field of research

Q Source: https://economictimes.indiatimes.com/news/politics-and-nation/india-excludes-pakistan-from-a-saarc -initiative/articleshow/62345991.cms

http://nkn.gov.in/about-us

offline.insightsias.com 41 © Insights Active Learning | All rights reserved - 554. You may not reproduce, distribute or exploit the contents in any form without written permission by copyright owner. Copyright infringers may face civil and criminal liability TEST - 20 (REVISION CUM Total Marks 200 MOCK TEST - 5) :

36 The Union Cabinet has approved the construction of Asia's longest bi-directional Zojila Pass tunnel. This will facilitate all-weather connectivity between

A. Srinagar and Leh B. Zanskar and Nathu La C. Ladakh and Xinjiang D. Siachin Glacier and Far eastern side of the Indo-China border

Correct Answer : A

Answer Justification :

Justification: The tunnel is being built at an estimated cost of Rs 6,089 crore (please ignore the figure mentioned in the image below), which will reduce the travel time between Srinagar and Leh to 15 minutes from the current 3.5 hours.

Zojila pass is situated at an altitude of 11,578 feet on Srinagar-Kargil-Leh National Highway which remains closed during winters (December to April) due to heavy snowfall and avalanches cutting off Leh-Ladakh region from Kashmir.

NAVEEN KUMAR N S - [email protected] - 9632470826

Q Source: https://economictimes.indiatimes.com/news/politics-and-nation/cabinet-approves-zojila-tunnel-projec t-in-jammu-and-kashmir/articleshow/62351425.cms

offline.insightsias.com 42 © Insights Active Learning | All rights reserved - 554. You may not reproduce, distribute or exploit the contents in any form without written permission by copyright owner. Copyright infringers may face civil and criminal liability TEST - 20 (REVISION CUM Total Marks 200 MOCK TEST - 5) :

37 Consider the following statements. #562 1. The work of river in the lower course is mainly Erosional rather than depositional due to the volume of water being carried by it. 2. When the river flows normally its bed is raised through accumulation of deposits.

Which of the above is/are correct? A. 1 only B. 2 only C. Both 1 and 2 D. None

Correct Answer : B

Answer Justification :

Justification: Statement 1: River slows down in the lower course which allows the sediments to settle down. This is opposite to the river’s action in the upper courses where the river mainly erodes.

Statement 2: As the debris or silt is deposited, it reduces the depth available for the water to flow, and thus the bed is raised.

Floodplains are created as a result of both erosion and deposition.

Material is also deposited on the sides forming raised banks called Levees.

When the river reaches the sea, the fine materials it has not dropped yet are deposited at its mouth, forming a fan shaped alluvial area called a delta.

Q Source: Chapter 5: Goh Cheng Leong - Certificate Physical and Human Geography

NAVEEN KUMAR N S - [email protected] - 9632470826

offline.insightsias.com 43 © Insights Active Learning | All rights reserved - 554. You may not reproduce, distribute or exploit the contents in any form without written permission by copyright owner. Copyright infringers may face civil and criminal liability TEST - 20 (REVISION CUM Total Marks 200 MOCK TEST - 5) :

38 The main roles and functions of the National Afforestation and Eco-Development Board (NAEB) include 1. Financing and overseeing the compensatory afforestation work 2. Rehabilitating and resettling communities displaced by deforestation for developmental works 3. Sponsor research and extension of research findings to disseminate new technologies for the regeneration and development of degraded forest

Select the correct answer using the codes below. A. 1 and 2 only B. 3 only C. 1 and 3 only D. 2 and 3 only

Correct Answer : B

Answer Justification :

Justification: Set up in 1992, the detailed role and functions of the NAEB are to:

Evolve mechanisms for ecological restoration of degraded forest areas and adjoining lands through systematic planning and implementation, in a cost effective manner;

Restore through natural regeneration or appropriate intervention the forest cover in the country for ecological security and to meet the fuelwood, fodder and other needs of the rural communities;

Restore fuelwood, fodder, timber and other forest produce on the degraded forest and adjoining lands in order to meet the demands for these items;

Sponsor research and extension of research findings to disseminate new and proper NAVEENtechnologies KUMAR for the N regenerationS - [email protected] and development of degraded forest- 9632470826 areas and adjoining lands;

Create general awareness and help foster people's movement for promoting afforestation and eco-development with the assistance of voluntary agencies, non-government organisations, Panchayati Raj institutions and others and promote participatory and sustainable management of degraded forest areas and adjoining lands;

Coordinate and monitor the Action Plans for afforestation, tree planting, ecological restoration and eco-development; and

Undertake all other measures necessary for promoting afforestation, tree planting, ecological restoration and eco-development activities in the country. offline.insightsias.com 44 © Insights Active Learning | All rights reserved - 554. You may not reproduce, distribute or exploit the contents in any form without written permission by copyright owner. Copyright infringers may face civil and criminal liability TEST - 20 (REVISION CUM Total Marks 200 MOCK TEST - 5) :

Q Source: From MoEFCC Website redirect - http://naeb.nic.in/

NAVEEN KUMAR N S - [email protected] - 9632470826

offline.insightsias.com 45 © Insights Active Learning | All rights reserved - 554. You may not reproduce, distribute or exploit the contents in any form without written permission by copyright owner. Copyright infringers may face civil and criminal liability TEST - 20 (REVISION CUM Total Marks 200 MOCK TEST - 5) :

39 The situation of fiscal deficit necessarily implies that 1. The Gross Domestic Product (GDP) is higher than Gross National Product (GNP). 2. The government is borrowing more from abroad than it is borrowing from domestic sources.

Which of the above is/are correct? A. 1 only B. 2 only C. Both 1 and 2 D. None

Correct Answer : D

Answer Justification :

Justification: Statement 1 and 2: These both are external situations, for e.g. in case of higher GDP than GNP, more income is being generated abroad by Indian nationals than being generated domestically by foreigners working or (investing) in India.

Moreover, a fiscal deficit situation may not require external borrowing to bridge the deficit at all, so statement 2 is also incorrect.

What this indicates is that the government is spending beyond its means. Basically, receipts are all forms of money accruing to the government, be it income or borrowings).

Fiscal deficit may be shown in the quantitative form (i.e., the total currency value of the deficit) or in the percentage form of the GDP for that particular year (percentage of GDP).

India has been a country of not only regular but higher fiscal deficits.

Moreover, the composition of its fiscal deficit has been more prone to criticism because it NAVEENalways KUMAR contains a Nrevenue S - [email protected] deficit component. - 9632470826

Q Source: Chapter 18: Indian Economy: Ramesh Singh

offline.insightsias.com 46 © Insights Active Learning | All rights reserved - 554. You may not reproduce, distribute or exploit the contents in any form without written permission by copyright owner. Copyright infringers may face civil and criminal liability TEST - 20 (REVISION CUM Total Marks 200 MOCK TEST - 5) :

40 Which of these can be found at Ajanta Caves? 1. Boddhisatva Images 2. Mahaparinibbana image of Buddha 3. Tales from Avadanas

Select the correct answer using the codes below. A. 1 and 2 only B. 2 and 3 only C. 1 and 3 only D. 1, 2 and 3

Correct Answer : D

Answer Justification :

Justification: The themes of the paintings are the events from the life of the Buddha, the Jatakas and the Avadanas. Some paintings such as Simhala Avadana, Mahajanaka Jataka and Vidhurpundita Jataka cover the entire wall of the cave.

The other important paintings are the famous Padmapani and Vajrapan.

Figures in these caves are painted with considerable naturalism and there is no over-stylisation.

Q Source: Chapter 4: 11th NCERT: An Introduction to Indian Arts

NAVEEN KUMAR N S - [email protected] - 9632470826

offline.insightsias.com 47 © Insights Active Learning | All rights reserved - 554. You may not reproduce, distribute or exploit the contents in any form without written permission by copyright owner. Copyright infringers may face civil and criminal liability TEST - 20 (REVISION CUM Total Marks 200 MOCK TEST - 5) :

41 The Directive Principles of State Policy (DPSP) provide for 1. Fundamental principles of governance 2. Constitutional rights and legal entitlements of marginalized sections of society

Which of the above is/are correct? A. 1 only B. 2 only C. Both 1 and 2 D. None

Correct Answer : A

Answer Justification :

Justification: The constitution itself mentions that Directive Principles of State Policy (DPSP) are fundamental to the governance of India, as they provides for the essential principles like equity, fairness, welfare orientation etc. So, 1 is correct.

2 will be incorrect because DPSP only gives guidance to the state in the formation of policies. The rights and entitlements can only be provided for in a law enacted by the legislature which gives force to the provisions of DPSP.

Q Source: Revision: Indian Polity: M Laxmikanth

NAVEEN KUMAR N S - [email protected] - 9632470826

offline.insightsias.com 48 © Insights Active Learning | All rights reserved - 554. You may not reproduce, distribute or exploit the contents in any form without written permission by copyright owner. Copyright infringers may face civil and criminal liability TEST - 20 (REVISION CUM Total Marks 200 MOCK TEST - 5) :

42 The main functions of the International Monetary Fund (IMF) include 1. To promote exchange rate stability and orderly exchange arrangements 2. To assist member countries by temporarily providing financial resources to correct mal-adjustment in their balance of payments (BoPs) 3. To serve as the regulator of international lending and borrowing from major international financial institutions

Select the correct answer using the codes below. A. 1 and 2 only B. 2 and 3 only C. 1 and 3 only D. 1, 2 and 3

Correct Answer : A

Answer Justification :

Justification: Statement 3: There is no international institution that regulates such lending and borrowing. World bank is one of the largest lenders but it is only a lender, does not regulate the process.

Statement 1 and 2: IMF was set up in 1944 with the main functions as exchange rate regulation, purchasing short-term foreign currency liabilities of the member nations from around the world, allotting special drawing rights (SDRs) to the member nations and the most important one as the bailor to the member economies in the situation of any BoP crisis.

Other major functions of the IMF are to facilitate international monetary cooperation and to assist in the establishment of a multilateral system of payments and the elimination of foreign exchange restrictions.

Q Source: Revision: Indian Economy: Ramesh Singh

NAVEEN KUMAR N S - [email protected] - 9632470826

offline.insightsias.com 49 © Insights Active Learning | All rights reserved - 554. You may not reproduce, distribute or exploit the contents in any form without written permission by copyright owner. Copyright infringers may face civil and criminal liability TEST - 20 (REVISION CUM Total Marks 200 MOCK TEST - 5) :

43 Which of the following fall under the Peninsular block of India? 1. Karbi Anglong Plateau 2. Rajmahal Hills 3. Meghalaya Plateau

Select the correct answer using the codes below: A. 1 and 2 only B. 2 and 3 only C. 1 only D. 1, 2 and 3

Correct Answer : D

Answer Justification :

Learning: The northern boundary of the Peninsular Block may be taken as an irregular line running from Kachchh along the western flank of the Aravali Range near Delhi and then roughly parallel to the Yamuna and the Ganga as far as the Rajmahal Hills and the Ganga delta.

Apart from these, the Karbi Anglong and the Meghalaya Plateau in the northeast and Rajasthan in the west are also extensions of this block. The northeastern parts are separated by the Malda fault in West Bengal from the Chotanagpur plateau. In Rajasthan, the desert and other desert–like features overlay this block.

Q Source: Revision: Chapter 2: 11th NCERT: India Physical Geography

NAVEEN KUMAR N S - [email protected] - 9632470826

offline.insightsias.com 50 © Insights Active Learning | All rights reserved - 554. You may not reproduce, distribute or exploit the contents in any form without written permission by copyright owner. Copyright infringers may face civil and criminal liability TEST - 20 (REVISION CUM Total Marks 200 MOCK TEST - 5) :

44 Old Stone Age sites have been an important topic of cultural studies in the Indian subcontinent. Which of the following general observations about them is CORRECT? 1. These sites are generally located near water sources. 2. Most of these sites are concentrated in Southern India with the exception of Siwalik hills and Bhimbetka.

Which of the above is/are correct? A. 1 only B. 2 only C. Both 1 and 2 D. None

Correct Answer : A

Answer Justification :

Justification: Statement 1: This may be because water sources were the lifeline of the Old stone age people and thus the communities lived and hunted close to water sources.

Statement 2: This is incorrect. Several rock shelters and caves used by the Paleolithic people are scattered across the subcontinent.

Some of the famous sites of Old Stone Age in India are:

The Soan valley and Potwar Plateau on the northwest India.

The Siwalik hills on the north India.

Bhimpetka in Madhya Pradesh.

NAVEENAdamgarh KUMAR hill in Narmada N S - [email protected] valley. - 9632470826

Kurnool in Andhra Pradesh and

Attirampakkam near Chennai.

Q Source: Revision: 11th NCERT: Introduction to Indian Arts

offline.insightsias.com 51 © Insights Active Learning | All rights reserved - 554. You may not reproduce, distribute or exploit the contents in any form without written permission by copyright owner. Copyright infringers may face civil and criminal liability TEST - 20 (REVISION CUM Total Marks 200 MOCK TEST - 5) :

45 How does the VAT system of taxation reduce the ‘cascading effect’ of taxes? 1. it avoids imposition of tax upon tax. 2. The system insists on single point tax collection.

Which of the above is/are correct? A. 1 only B. 2 only C. Both 1 and 2 D. None

Correct Answer : A

Answer Justification :

Justification: Production of goods or services is nothing our stages of value additions where production of goods is done by the industrialists or manufacturers.

From production to the level of sale, there are many points where value is added in all goods.

VAT method of tax collection is different from the non-VAT method in the sense that it is imposed and collected at different points of value addition chain, i.e., multi-point tax collection.

That is why there is no chance of imposing tax upon tax which takes place in the non-VAT method—single point tax collection.

This is why VAT does not have a cascading effect’ on the prices of goods it does not increase inflation— and is therefore suitable for an economy like India.

Q Source: Chapter 17: Indian Economy: Ramesh Singh

NAVEEN KUMAR N S - [email protected] - 9632470826

offline.insightsias.com 52 © Insights Active Learning | All rights reserved - 554. You may not reproduce, distribute or exploit the contents in any form without written permission by copyright owner. Copyright infringers may face civil and criminal liability TEST - 20 (REVISION CUM Total Marks 200 MOCK TEST - 5) :

46 Consider the following about BioCNG. 1. It is a purified form of biogas with a major content of butane and propane. 2. While it cannot be used as a vehicle fuel, it yields a good calorific value as a domestic fuel. 3. It can be obtained from pure cellulosic biomass. 4. Increased production of BioCNG in India will help reduce import dependency on petroleum products.

Select the correct answer using the codes below. A. 1, 2 and 4 only B. 3 and 4 only C. 1, 2 and 3 only D. 2, 3 and 4 only

Correct Answer : B

Answer Justification :

Justification: Transport sector is one of the major oil consuming sectors; it consumes 51% of the final oil consumption (International Energy Agency, 2010).

The use of biogas-‐CNG blend in CNG vehicles is not possible without upgrading the quality of the biogas. Biomethane is upgraded biogas which can be used in CNG vehicles as a fuel.

Statement 1: Bio-CNG is a purified form of biogas with over 95% pure methane gas. It is similar to natural gas in its composition (97% methane) and energy potential. While natural gas is a fossil fuel, bio-CNG is a renewable form of energy produced from agricultural and food waste. Bio-CNG is being looked at as an environment-friendly alternative to diesel.

Statement 2: This Bio CNG can be used to power automobiles with the same efficiency as fossil fuel CNG and is now dispensed under the brand name AgroGasTM.

Bio CNG has a high calorific value and can be used in blast furnaces; also it can be converted into electricity NAVEEN KUMAR N S - [email protected] - 9632470826 Statement 3:

offline.insightsias.com 53 © Insights Active Learning | All rights reserved - 554. You may not reproduce, distribute or exploit the contents in any form without written permission by copyright owner. Copyright infringers may face civil and criminal liability TEST - 20 (REVISION CUM Total Marks 200 MOCK TEST - 5) :

Statement 4: The cost of production of 1kg of bio-CNG could be Rs15-20, cheaper than CNG, petrol and diesel. Besides, it will help in reducing the country’s import of diesel up to 50%. It is also pollution free.

Q Source: http://mnre.gov.in/file-manager/akshay-urja/november-december-2016/EN/Images/31-33.pdf

http://www.climatetechwiki.org/technology/biomethane-cng

http://www.insightsonindia.com/2017/12/30/insights-daily-current-affairs-30-december-2017/

NAVEEN KUMAR N S - [email protected] - 9632470826

offline.insightsias.com 54 © Insights Active Learning | All rights reserved - 554. You may not reproduce, distribute or exploit the contents in any form without written permission by copyright owner. Copyright infringers may face civil and criminal liability TEST - 20 (REVISION CUM Total Marks 200 MOCK TEST - 5) :

47 Nations bordering Russia include 1. Mongolia 2. Uzbekistan 3. China 4. Turkmenistan

Select the correct answer using the codes below. A. 1 and 3 only B. 1, 2 and 3 only C. 2 and 4 only D. 1, 2, 3 and 4

Correct Answer : A

Answer Justification :

Learning:

NAVEEN KUMAR N S - [email protected] - 9632470826

Q Source: Map-based: Central Asia and East Asia

offline.insightsias.com 55 © Insights Active Learning | All rights reserved - 554. You may not reproduce, distribute or exploit the contents in any form without written permission by copyright owner. Copyright infringers may face civil and criminal liability TEST - 20 (REVISION CUM Total Marks 200 MOCK TEST - 5) :

48 Non-tax revenue receipts of the Government of India do NOT include

A. Profits and dividends which the government gets from its public sector undertakings (PSUs) B. Loans recovered by the Government of India out of all loans forwarded by it C. External grants received by the Government D. None of the above is a part of non-tax revenue receipts.

Correct Answer : B

Answer Justification :

Learning: These are the regular accruals to the government based on the revenue that does not come from tax.

In India they are:

Profits and dividends which the government gets from its public sector undertakings (PSUs)

Interest, received by the government out of all loans forwarded by it, be it inside the country (i.e., internal lending) or outside the country (i.e., external lending).

It means this income might be in both domestic and foreign currencies.

Pascal services also generate incomes for the government. i.e., currency printing, stamp printing, coinage, medals, minting, etc.

General Services also earn money for the government as the power distribution, irrigation, banking, insurance, community services, etc.

NAVEEN KUMAR N S - [email protected] - 9632470826 Fees, Penalties and Fines received by the government.

Grants which the governments receives— it is always external in the case of the Central Government and internal in the case of state governments.

Q Source: Chapter 18: Indian Economy: Ramesh Singh

offline.insightsias.com 56 © Insights Active Learning | All rights reserved - 554. You may not reproduce, distribute or exploit the contents in any form without written permission by copyright owner. Copyright infringers may face civil and criminal liability TEST - 20 (REVISION CUM Total Marks 200 MOCK TEST - 5) :

49 Which of these nations has recently announced its plans to launch a lunar probe in 2018 to achieve the world’s first soft landing on the far side of the moon?

A. USA B. Russia C. China D. India

Correct Answer : C

Answer Justification :

Learning: The mission is called Chang’e 4 project.

According to experts, landing on the far side of the moon is undoubtedly one of the most challenging missions ever launched by any of the world’s superpowers.

The far side of the moon known as ‘South Pole-Aitken Basin’ still remains a mystery among space scientists and by sending a probe there, China will outdo the historical achievements of the US and USSR.

Communication difficulties will be the main problem faced by the Chinese team as they try to land on the other side of the moon due to the absence of a transmitting medium.

China is expected to consider using options like radio telescopes to communicate in the absence of a transmitting medium.

Q Source: http://indianexpress.com/article/technology/science/china-plans-to-become-first-country-to-land-on-d ark-side-of-the-moon-4446976/

NAVEEN KUMAR N S - [email protected] - 9632470826

offline.insightsias.com 57 © Insights Active Learning | All rights reserved - 554. You may not reproduce, distribute or exploit the contents in any form without written permission by copyright owner. Copyright infringers may face civil and criminal liability TEST - 20 (REVISION CUM Total Marks 200 MOCK TEST - 5) :

50 Revenue deficit is problematic in Government budget because

A. The government cannot borrow further for spending on developmental programmes. B. It is not a sustainable financial position. C. This indicates rampant tax evasion and tax avoidance. D. It leads to hyperinflation in both the short-term and long-term.

Correct Answer : B

Answer Justification :

Justification: Option A: The government can still borrow leading to further expansion of the fiscal deficit. This has been the GOI’s budget position since decades.

Option B: This is because the government is spending more than it is earning. It is not maintainable in the long-run, the extra spending will somehow has to be financed else it will lead to a debt spiral.

Option C: It is not necessary because even a better managed economy in terms of taxation can have poor revenue yield if the revenue resources are not significant. But, if tax evasion is rampant, it will surely deplete the revenue resources and add to the revenue deficit.

Option D: This is only a possibility and that too when the revenue deficit is very high and the government ignores creation of supply chain and production capacity in the economy. This is because due to high revenue expenditure, demand will increase but supply will not increase in the same proportion.

Q Source: Chapter 18: Indian Economy: Ramesh Singh

NAVEEN KUMAR N S - [email protected] - 9632470826

offline.insightsias.com 58 © Insights Active Learning | All rights reserved - 554. You may not reproduce, distribute or exploit the contents in any form without written permission by copyright owner. Copyright infringers may face civil and criminal liability TEST - 20 (REVISION CUM Total Marks 200 MOCK TEST - 5) :

51 MARPOL Convention is related to

A. Prevention of Pollution from Ships B. International Civil Aviation Protocol C. Transnational mining regulation D. Use of water of transnational rivers

Correct Answer : A

Answer Justification :

Learning: It is also known as International Convention for the Prevention of Pollution from Ships (MARPOL Convention).

It is the main international convention that deals with this subject. It was adopted in 1973 and entered into force in 1983.

Its biodiversity relevance stems from the highly detrimental effect that pollution can have on marine organisms, therefore by preventing the pollution of the sea the Convention contributes towards the maintenance of marine biodiversity.

Q Source: http://www.biodiversitya-z.org/

NAVEEN KUMAR N S - [email protected] - 9632470826

offline.insightsias.com 59 © Insights Active Learning | All rights reserved - 554. You may not reproduce, distribute or exploit the contents in any form without written permission by copyright owner. Copyright infringers may face civil and criminal liability TEST - 20 (REVISION CUM Total Marks 200 MOCK TEST - 5) :

52 The New Urban Agenda is an important outcome document agreed upon at the

A. Paris Climate Conference, 2016 B. Adis Ababa Climate Conference, 2014 C. Habitat III cities conference, 2016 D. 2015 United Nations Climate Change Conference

Correct Answer : C

Answer Justification :

Learning: It will guide the efforts around urbanization of a wide range of actors — nation states, city, United Nations programmes and civil society — for the next 20 years laying the groundwork for policies.

Habitat III, the United Nations Conference on Housing and Sustainable Urban Development, took place in Quito, Ecuador, from 17 – 20 October 2016.

Habitat III was one of the first United Nations global summits after the adoption of the Post-2015 Development Agenda.

It gave the opportunity to open discussions on important urban challenges and questions, such as how to plan and manage cities, towns and villages for sustainable development.

Q Source http://www.un.org/sustainabledevelopment/blog/2016/10/newurbanagenda/

NAVEEN KUMAR N S - [email protected] - 9632470826

offline.insightsias.com 60 © Insights Active Learning | All rights reserved - 554. You may not reproduce, distribute or exploit the contents in any form without written permission by copyright owner. Copyright infringers may face civil and criminal liability TEST - 20 (REVISION CUM Total Marks 200 MOCK TEST - 5) :

53 Why do several Indian economists caution against printing currency? 1. It is inflationary in that it may not be accompanied with simultaneous expansion in productive capacity of the economy. 2. It requires Gold backing which pushes the price of Gold and thus the import bill of India.

Which of the above is/are correct? A. 1 only B. 2 only C. Both 1 and 2 D. None

Correct Answer : A

Answer Justification :

Justification: Statement 1: When we print currency, we inject additional demand in the economic system, and this increased demand need not be accompanied by increased supply of goods. This may be inflationary in most cases.

It brings in regular pressure and obligation on the government for upward revision in wages and salaries of government employees— ultimately increasing the government expenditures necessitating further printing of currency and further inflation—a vicious cycle into which economies entangle themselves.

Hence, printing currency is the last resort for the government in managing its deficit.

Statement 2: The system of gold backing has been repealed, and the RBI manages printing of currency. This is because the additional currency is a liability on RBI and it must ensure that it has sufficient reserves to clear this liability.

Q Source: Chapter 18: Indian Economy: Ramesh Singh

NAVEEN KUMAR N S - [email protected] - 9632470826

offline.insightsias.com 61 © Insights Active Learning | All rights reserved - 554. You may not reproduce, distribute or exploit the contents in any form without written permission by copyright owner. Copyright infringers may face civil and criminal liability TEST - 20 (REVISION CUM Total Marks 200 MOCK TEST - 5) :

54 Conditions that favour the formation of deltas include 1. Coasts without shelters with a large tidal variation 2. Frequent water diversions from the main river that forms the delta 3. Presence of Continental shelf

Select the correct answer using the codes below. A. 1 and 2 only B. 3 only C. 2 and 3 only D. 1 and 3 only

Correct Answer : B

Answer Justification :

Justification: Conditions Favourable for Delta Formation

Active erosion of the river in its upper course to provide extensive gravel, sand and silt to be eventually deposited as deltas.

The coast should be sheltered preferably tideless, else delta will be washed away.

There should be no strong current running at right angle to the river mouth, as it can wash away the sediments.

Any large lake in the way or river course can filter off the sediments, thus unfavourable for delta formation.

The sea should be shallow adjoining the delta as the sediments will disappear in the deep NAVEENwaters KUMAR of the sea. NThis S is - [email protected] a continental shelf is preferred. - 9632470826

Q Source: Chapter 5: Goh Cheng Leong - Certificate Physical and Human Geography

offline.insightsias.com 62 © Insights Active Learning | All rights reserved - 554. You may not reproduce, distribute or exploit the contents in any form without written permission by copyright owner. Copyright infringers may face civil and criminal liability TEST - 20 (REVISION CUM Total Marks 200 MOCK TEST - 5) :

55 Consider the following statements. Assertion (A): Adding limestone to the soil frees up some of the soil minerals making them available for absorption into the plant. Reason (R): Calcium carbonate content of the limestone is capable of neutralising some of the acid in the soil.

In the context of the above, which of these is correct? A. A is correct, and R is an appropriate explanation of A. B. A is correct, but R is not an appropriate explanation of A. C. A is correct, but R is incorrect. D. Both A and R are incorrect.

Correct Answer : A

Answer Justification :

Justification: Limestone deposits with a calcium carbonate content around 90% can be quarried to be used by the farming sector. By crushing the limestone, various grades (based on particle size) can be produced. These products are marketed as ‘agricultural lime.

Soil acidity is one of the factors that can influence plant growth and can seriously limit crop production.

By spreading agricultural lime onto the paddock or soil, the calcium carbonate content of the limestone is capable of neutralising some of the acid in the soil. This also has the effect of freeing up some of the soil minerals, such as phosphates, and making them available for absorption into the plant.

Q Source: Additional Research: Chapter on Agriculture: 8th NCERT Geography

NAVEEN KUMAR N S - [email protected] - 9632470826

offline.insightsias.com 63 © Insights Active Learning | All rights reserved - 554. You may not reproduce, distribute or exploit the contents in any form without written permission by copyright owner. Copyright infringers may face civil and criminal liability TEST - 20 (REVISION CUM Total Marks 200 MOCK TEST - 5) :

56 Which of the following is NOT an official language of the United Nations?

A. Chinese B. Arabic C. French D. Hindi

Correct Answer : D

Answer Justification :

Justification: There are six official languages of the UN. These are Arabic, Chinese, English, French, Russian and Spanish. These languages are used at meetings of various UN organs, particularly the General Assembly, the Economic and Social Council, and the Security Council.

India is pushing forward its proposal to include Hindi as one of the official languages of the United Nations.

Each representative of a country may speak in any one of these six languages, or may speak in any language and provide interpretation into one of the six official languages.

The procedure for getting any language recognized as one of the Official Languages of the UN involves obtaining approval of the General Assembly.

In addition, recognizing any language as the official language of the UN entails a substantial increase in the expenditure of the UN necessitating an enhanced contribution by every member country, which is why most of the members remain reluctant to support such a proposal.

NAVEENLearning: ThereKUMAR is a reason N S why - [email protected] the GOI is pushing for inclusion of Hindi - 9632470826 as an official language.

According to the estimates around 340 million to 500 million speak, and as many as 800 million people understand Hindi language.

Outside India there are countries like Nepal South Africa, Mauritius, the United Kingdom, the United States, Yemen, and Uganda where a significant number people speak Hindi.

Also, the government believes that at a time when Indian economy is shining and the country is emerging as next global power it is the best time for the country to promote its national language and get it recognised as an official language of United Nations.

Q Source: http://www.insightsonindia.com/2018/01/04/insights-daily-current-affairs-04-january-2018/ offline.insightsias.com 64 © Insights Active Learning | All rights reserved - 554. You may not reproduce, distribute or exploit the contents in any form without written permission by copyright owner. Copyright infringers may face civil and criminal liability TEST - 20 (REVISION CUM Total Marks 200 MOCK TEST - 5) :

57 Why are estuaries considered favourable for situating ports? 1. Estuaries have an open connection to the Sea or Ocean. 2. Estuaries have considerable depth of water for anchoring and stationing ships.

Which of the above is/are correct? A. 1 only B. 2 only C. Both 1 and 2 D. None

Correct Answer : C

Answer Justification :

Justification: An estuary is a partially enclosed coastal body of brackish water with one or more rivers or streams flowing into it, and with a free connection to the open sea.

They are an important part of the shipping industry because there are many industrial ports located in estuaries due its depth of water and connection with open sea.

As against estuaries, deltas are not preferred for coastal shipping because deltas do not offer the depth of water required for shipping and may not offer the required connection to the open sea.

Q Source: Chapter 5: Goh Cheng Leong - Certificate Physical and Human Geography

NAVEEN KUMAR N S - [email protected] - 9632470826

offline.insightsias.com 65 © Insights Active Learning | All rights reserved - 554. You may not reproduce, distribute or exploit the contents in any form without written permission by copyright owner. Copyright infringers may face civil and criminal liability TEST - 20 (REVISION CUM Total Marks 200 MOCK TEST - 5) :

58 Which of the following events were involved in and around the Tripuri Crisis of the Indian National Congress (INC)? 1. All-India Muslim League forming a separate unit in the Indian National Congress (INC). 2. British banning the Indian National Congress (INC) 3. Contentions related to leadership of the INC

Select the correct answer using the codes below. A. 1 and 2 only B. 1 and 3 only C. 3 only D. 2 and 3 only

Correct Answer : C

Answer Justification :

Justification: In 1939, S.C. Bose contended for the presidentship of Congress and he defeated Pattabhi Sitaramaiya, candidate of Gandhi.

Gandhi declared the defeat of Pattabhi as his own defeat.

Ultimately in April 1939, S.C. Bose resigned and Dr. Rajendra Prasad was made new President.

In May 1939, S.C. Bose founded a new party 'Forward Block' at Makur, Unnao.

In August 1939, S.C. Bose was removed from the post of President of All India Congress committee and Bengal Congress Committee and was declared unfit for any post in Congress for the next 3 years.

This was the Tripuri crisis of the INC.

Q Source: Additional research: 12th NCERT: Our-pasts III

NAVEEN KUMAR N S - [email protected] - 9632470826

offline.insightsias.com 66 © Insights Active Learning | All rights reserved - 554. You may not reproduce, distribute or exploit the contents in any form without written permission by copyright owner. Copyright infringers may face civil and criminal liability TEST - 20 (REVISION CUM Total Marks 200 MOCK TEST - 5) :

59 Elections in country X will be “Democratic” if

A. Country X has a written constitution. B. Country X has a very large voter and elector base. C. If an independent and powerful body supervises elections in Country X. D. Country X has a large number of political parties.

Correct Answer : C

Answer Justification :

Justification: there are several conditions that make an election democratic:

There should be a large number of players involved and each one of them should have a fair chance of winning.

Moreover, the voting power should be exercised by the people in the presence of an independent body.

Also, the verdict as given by the people must be accepted peacefully by the losing parties.

Option A: Syria has had a constitution for long and yet its elections could not have been called democratic.

Option B: Merely a large voter base does not ensure successful and democratic elections. For e.g. Iraq may have a large voter base, many agencies alleged that its 2003 elections were not democratic. Malpractices, rigging etc can take place in countries with large voter base. In such cases, elections will not be fair and democratic.

Q Source: Revision: 9th NCERT: Democratic Polity

NAVEEN KUMAR N S - [email protected] - 9632470826

offline.insightsias.com 67 © Insights Active Learning | All rights reserved - 554. You may not reproduce, distribute or exploit the contents in any form without written permission by copyright owner. Copyright infringers may face civil and criminal liability TEST - 20 (REVISION CUM Total Marks 200 MOCK TEST - 5) :

60 In taxation, horizontal equity implies

A. Similarly situated people pay the same taxes. B. Better off people pay more taxes. C. Everyone pays the same amount of tax. D. Taxation is independent of income levels.

Correct Answer : A

Answer Justification :

Learning: Taxation is based on the idea of Fairness. Though fairness (i.e., the first criteria of a good tax system) is not always easy to define, economists suggested inclusion of two elements in the tax system to make it fair namely, horizontal equity and vertical equity.

Individuals in identical or similar situations paying identical or similar taxes is known as horizontal equity. When ‘better off people pay more taxes it is known as vertical equity. When we combine both equities, we each closer to the Indian taxation system.

Q Source: Chapter 17: Indian Economy: Ramesh Singh

NAVEEN KUMAR N S - [email protected] - 9632470826

offline.insightsias.com 68 © Insights Active Learning | All rights reserved - 554. You may not reproduce, distribute or exploit the contents in any form without written permission by copyright owner. Copyright infringers may face civil and criminal liability TEST - 20 (REVISION CUM Total Marks 200 MOCK TEST - 5) :

61 Consider the following statements. 1. It is India's largest inland salt lake. 2. It is geographically a land locked river basin. 3. It has been designated as a Ramsar site.

The above refer to? A. Bhimtal Lake B. Bhitarkanika Lake C. Sambhar Lake D. Shakambari Lake

Correct Answer : C

Answer Justification :

Learning: Sambhar has been designated as a Ramsar site (recognized wetland of international importance) because the wetland is a key wintering area for tens of thousands of flamingos and other birds that migrate from northern Asia.

The specialized algae and bacteria growing in the lake provide striking water colours and support the lake ecology that, in turn, sustains the migrating waterfowl.

It is not part of the Ganga river basin area and is geographically a separate land locked river basin.

It is the source of most of Rajasthan's salt production.

Q Source: Major Wetlands of India

NAVEEN KUMAR N S - [email protected] - 9632470826

offline.insightsias.com 69 © Insights Active Learning | All rights reserved - 554. You may not reproduce, distribute or exploit the contents in any form without written permission by copyright owner. Copyright infringers may face civil and criminal liability TEST - 20 (REVISION CUM Total Marks 200 MOCK TEST - 5) :

62 In the Gupta administration, Sandivigraha were those officials who

A. Assisted the king in maintaining a close contact with the provincial administration B. Looked after the city administration C. Were princes and later became ministers of provinces D. Administered foreign affairs

Correct Answer : D

Answer Justification :

Learning: A high official called was mentioned in the Gupta inscriptions known as Sandivigraha, most probably minister for foreign affairs.

The king was assisted in his administration by a council consisting of a chief minister, a Senapati or commanderin- chief of the army and other important officials.

The king maintained a close contact with the provincial administration through a class of officials called Kumaramatyas and Ayuktas.

Provinces in the Gupta Empire were known as Bhuktis and provincial governors as Uparikas. They were mostly chosen from among the princes.

Q Source: Revision: Chapter 9: 11th Std. TamilNadu History Textbook

NAVEEN KUMAR N S - [email protected] - 9632470826

offline.insightsias.com 70 © Insights Active Learning | All rights reserved - 554. You may not reproduce, distribute or exploit the contents in any form without written permission by copyright owner. Copyright infringers may face civil and criminal liability TEST - 20 (REVISION CUM Total Marks 200 MOCK TEST - 5) :

63 Consider the following matches of Buddhist Monasteries with their locations. 1. : Located in a Valley of Northern India 2. Kardang Monastery : Most populated monastery in Southern India 3. Rumtek Monastery : Largest in

Select the correct answer using the codes below. A. 1 and 2 only B. 3 only C. 1 only D. 2 only

Correct Answer : C

Answer Justification :

Learning: Statement 1: It is located in the Tabo village of , Himachal Pradesh. It was founded in 996 CE in the Tibetan year of the Fire Ape.

Tabo is noted for being the oldest continuously operating Buddhist enclave in both India and the Himalayas. A large number of frescoes displayed on its walls depict tales from the Buddhist pantheon.

Statement 2: Kardang Monastery or Gompa is a famous monastery, and is the most important monastery the Lahaul valley, India. The monastery is a huge white building bedecked with prayer flags.

Statement 3: The monastery is currently the largest in Sikkim. It is home to the community of monks and where they perform the rituals and practices of the Karma lineage. A golden contains the relics of the 16th . Opposite that building is a college, Karma Shri Nalanda Institute for Higher Buddhist Studies.

Q Source: Additional Research: 11th NCERT: Introduction to Indian arts

NAVEEN KUMAR N S - [email protected] - 9632470826

offline.insightsias.com 71 © Insights Active Learning | All rights reserved - 554. You may not reproduce, distribute or exploit the contents in any form without written permission by copyright owner. Copyright infringers may face civil and criminal liability TEST - 20 (REVISION CUM Total Marks 200 MOCK TEST - 5) :

64 Consider the following about White Dwarfs. 1. They are the lightest forms of matter. 2. They can give an idea about the age of the Universe. 3. White dwarfs are the end product of black holes.

Select the correct answer using the codes below. A. 1 and 2 only B. 3 only C. 2 only D. 1, 2 and 3

Correct Answer : C

Answer Justification :

Justification: White dwarfs are one of the densest forms of matter, surpassed only by neutron stars and black holes. So, 1 is wrong.

A low or medium mass star (with mass less than about 8 times the mass of our Sun) will become a white dwarf. Stars that have a lot of mass may end their lives as black holes or neutron stars. So, 3 is wrong.

White dwarfs can tell us about the age of the Universe. If we can estimate the time it takes for a white dwarf to cool into a black dwarf, that would give us a lower limit on the age of the Universe and our galaxy. So, 2 is correct.

Q Source: https://imagine.gsfc.nasa.gov/science/objects/dwarfs1.html

NAVEEN KUMAR N S - [email protected] - 9632470826

offline.insightsias.com 72 © Insights Active Learning | All rights reserved - 554. You may not reproduce, distribute or exploit the contents in any form without written permission by copyright owner. Copyright infringers may face civil and criminal liability TEST - 20 (REVISION CUM Total Marks 200 MOCK TEST - 5) :

65 She is widely remembered for hoisting the Indian National Congress flag at the Gowalia Tank maidan in Bombay during the Quit India Movement, 1942 and participated in public processions during the Salt Satyagraha. She is

A. Begum Hazrat Mahal B. Annie Besant C. Kasturba Gandhi D. Aruna Asaf Ali

Correct Answer : D

Answer Justification :

Learning: She became an active member of Congress Party after marrying Asaf Ali and participated in public processions during the Salt Satyagraha.

In 1954, she helped form the National Federation of Indian Women, the women's wing of CPI but left the party in 1956 following Nikita Khrushchev's disowning of Stalin.

She received India's highest civilian award, the Bharat Ratna, posthumously in 1997. She was a member of the Congress Socialist Party

Q Source: Major Women leaders: INM

NAVEEN KUMAR N S - [email protected] - 9632470826

offline.insightsias.com 73 © Insights Active Learning | All rights reserved - 554. You may not reproduce, distribute or exploit the contents in any form without written permission by copyright owner. Copyright infringers may face civil and criminal liability TEST - 20 (REVISION CUM Total Marks 200 MOCK TEST - 5) :

66 Which of these tribes belong to the Andaman and Nicobar Islands? 1. Shompen 2. Jarawas 3. Onge 4. Sentinelese

Select the correct answer using the codes below. A. 1 and 2 only B. 3 only C. 2 and 4 only D. 1, 2, 3 and 4

Correct Answer : D

Answer Justification :

Justification: Shompen are the indigenous people of the interior of Great Nicobar Island.

The Shompen are a designated Scheduled Tribe and a PTG.

Not as well known as the Jarawa tribe of the Andaman islands, the Shompen are as precariously poised on the brink of extinction as the four other hunter-gatherer tribes (the Jarawa, the Andamanese, the Onge and the Sentinelese

The Shompen languages, of which there are at least two, are very little known, but appear to be unrelated to Nicobarese, an isolated group of Austroasiatic languages, and perhaps even to each other. They may constitute a language isolate.

We will cover details about other tribes in upcoming tests. They are important.

Q Source: Revision: 11th NCERT: Fundamentals of Human Geography: India

NAVEEN KUMAR N S - [email protected] - 9632470826

offline.insightsias.com 74 © Insights Active Learning | All rights reserved - 554. You may not reproduce, distribute or exploit the contents in any form without written permission by copyright owner. Copyright infringers may face civil and criminal liability TEST - 20 (REVISION CUM Total Marks 200 MOCK TEST - 5) :

67 Consider the following statements. 1. The Ministries/Departments of the Government are created by the President on the advice of the Prime Minister. 2. Each of the Ministries is assigned to a Minister by the President on the advice of the Prime Minister. 3. The rules for the allocation of business of the Government of India are made by the President.

Select the correct answer using the codes below. A. 1 and 2 only B. 3 only C. 2 and 3 only D. 1, 2 and 3

Correct Answer : D

Answer Justification :

Justification & Learning: The Government of India (Allocation of Business) Rules, 1961 are made by the President of India under Article 77 of the Constitution for the allocation of business of the Government of India. So, 3 is correct.

The Ministries/Departments of the Government are created by the President on the advice of the Prime Minister under these Rules. The business of the Government are transacted in the Ministries/Departments, Secretariats and offices (referred to as 'Department') as per the distribution of subjects specified in these Rules.

Each of the Ministries is assigned to a Minister by the President on the advice of the Prime Minister. Each department is generally under the charge of a Secretary to assist the Minister on policy matters and general administration.

Q Source: Additional Research: 10th NCERT: Democratic Politics

NAVEEN KUMAR N S - [email protected] - 9632470826

offline.insightsias.com 75 © Insights Active Learning | All rights reserved - 554. You may not reproduce, distribute or exploit the contents in any form without written permission by copyright owner. Copyright infringers may face civil and criminal liability TEST - 20 (REVISION CUM Total Marks 200 MOCK TEST - 5) :

68 In-situ initiatives beyond protected areas may include 1. On-farm agricultural biodiversity conservation targeted at traditional crop varieties and crop wild relatives 2. Genetic sanctuaries in natural wild surroundings 3. Preserving species in Botanical gardens

Select the correct answer using the codes below. A. 1 only B. 2 and 3 only C. 1 and 2 only D. 1, 2 and 3

Correct Answer : C

Answer Justification :

Justification: Statement 3: This is an example of ex-situ conservation because a botanical garden is not the natural habitat of those species.

In-situ measures are perceived as more holistic in their approach and allow the conservation of processes or habitats which can’t be protected through ex-situ measures (e.g. soil microbial processes, evolutionary processes, specific ecosystems such as coral reefs or species with highly specialised needs).

Other examples of in situ (natural site) conservation are:

Habitat restoration, recovery or rehabilitation;

Strategies for the sustainable use and management of biological resources;

Recovery programmes for nationally or sub-nationally threatened or endangered wild species; NAVEEN KUMAR N S - [email protected] - 9632470826

Genetic reserve conservation, ie. monitoring of genetic diversity in natural wild populations within a delineated area (known as genetic sanctuaries or gene management zones)

Control of threats to biodiversity such as invasive alien species, living modified organisms or over exploitation;

Preservation and maintenance of traditional knowledge and practices; and

Implementation of the regulatory, legislation, management or other frameworks needed to deliver the protection of species or habitats.

The Convention on Biological Diversity (CBD) recognizes the complementary role of both offline.insightsias.com 76 conservation strategies (in-situ and ex-situ). © Insights Active Learning | All rights reserved - 554. You may not reproduce, distribute or exploit the contents in any form without written permission by copyright owner. Copyright infringers may face civil and criminal liability TEST - 20 (REVISION CUM Total Marks 200 MOCK TEST - 5) :

Q Source: Related topics: Environment and Biodiversity

NAVEEN KUMAR N S - [email protected] - 9632470826

offline.insightsias.com 77 © Insights Active Learning | All rights reserved - 554. You may not reproduce, distribute or exploit the contents in any form without written permission by copyright owner. Copyright infringers may face civil and criminal liability TEST - 20 (REVISION CUM Total Marks 200 MOCK TEST - 5) :

69 Consider the following hills and their locations. 1. Shevroy Hills: Eastern Ghats 2. Cardamom Hills: Western Ghats 3. Anaimalai Hills: Garhjat Range

Select the correct answer using the codes below. A. 1 and 2 only B. 2 only C. 1 and 3 only D. 1, 2 and 3

Correct Answer : A

Answer Justification :

Justification: Statement 1: The Servarayans form part of the southern ranges of the Eastern Ghats System. it also represents the highest peak in southern part of the Eastern Ghats, with the Solaikaradu peak.

Statement 2: They conjoin the Anaimalai Hills to the northwest, the Palni Hills to the northeast and the Agasthyamalai Hills to the south as far as the Aryankavu. The crest of the hills forms the boundary between Kerala and Tamil Nadu. Anamudi in Eravikulam National Park, is the highest peak in Western Ghats and also the highest point in India south of the Himalayas

Statement 3: They form a southern portion of the Western Ghats. Anamala / Anaimalai Hills are south of where the Western Ghats are broken by the Palakkad Gap, which in turn is south of the Nilgiri Hills. They border the state of Kerala on the Southwest and the Cardamom Hills to the southeast.

Q Source: Additional Research: 11th NCERT: India Physical Geography

NAVEEN KUMAR N S - [email protected] - 9632470826

offline.insightsias.com 78 © Insights Active Learning | All rights reserved - 554. You may not reproduce, distribute or exploit the contents in any form without written permission by copyright owner. Copyright infringers may face civil and criminal liability TEST - 20 (REVISION CUM Total Marks 200 MOCK TEST - 5) :

70 Which of the following steps help prevent the spread of Influenza? 1. Getting anti-bacterial vaccination and drugs 2. Avoiding touching one’s eyes, nose or mouth 3. Early self-isolation of those having symptoms of Influenza

Select the correct answer using the codes below. A. 1 and 2 only B. 2 and 3 only C. 3 only D. 1 and 3 only

Correct Answer : B

Answer Justification :

Justification: Avian influenza, commonly called bird flu, is an infectious viral disease of birds with a tendency of causing large-scale outbreaks of serious disease. Although most influenza viruses do not infect humans, A(H5N1) and A(H7N9) have caused serious infections in people.

An outbreak of H5 Avian Influenza has been reported from Dasarahalli village from Bengaluru Urban District in Karnataka. On the directions of the Centre, the Karnataka Government has notified the outbreak and initiated the control and containment operations.

Statement 1: It is a viral disease, so anti-viral (not anti-bacterial) drugs may reduce the duration of viral replication and improve prospects of survival.

Statement 2 and 3: Personal protective measures like these can work:

Regular hand washing with proper drying of the hands

Good respiratory hygiene – covering mouth and nose when coughing or sneezing, using tissues and disposing of them correctly NAVEEN KUMAR N S - [email protected] - 9632470826

Early self-isolation of those feeling unwell, feverish and having other symptoms of influenza

Avoiding close contact with sick people

Travelers to countries and people living in countries with known outbreaks of avian influenza should, if possible, avoid poultry farms, contact with animals in live poultry markets, entering areas where poultry may be slaughtered, and contact with any.

Q Source: http://www.who.int/mediacentre/factsheets/avian_influenza/en/

Additional Research: http://www.insightsonindia.com/2018/01/06/insights-daily-current-affairs-06-january-2018/ offline.insightsias.com 79 © Insights Active Learning | All rights reserved - 554. You may not reproduce, distribute or exploit the contents in any form without written permission by copyright owner. Copyright infringers may face civil and criminal liability TEST - 20 (REVISION CUM Total Marks 200 MOCK TEST - 5) :

71 Which of the following regions of India is known as the Tiger Capital of the country?

A. Gir B. Nagpur C. Bandipur D. Nagarhole

Correct Answer : B

Answer Justification :

Learning: The central government is promoting Nagpur in Maharashtra as the tiger capital.

As per an official statement, Maharashtra has done exceptionally well in the growth of tigers- which has prompted the government to promote Nagpur, which is the central most city of the country.

Maharashtra has a tiger population of 169 according to latest census.

Nagpur and its nearby reserves in Maharashtra have 148 of these. In addition, the city was close to many tiger reserves in Madhya Pradesh like Kanha, Pench, and Satpuda.

There are many notified tiger parks and reserves in its close proximity.

It is promoted as the tiger capital of the world given the city's closeness with several tiger reserves and its infrastructure that can support a large tourist industry.

Q Source: Recent Tiger Census

NAVEEN KUMAR N S - [email protected] - 9632470826

offline.insightsias.com 80 © Insights Active Learning | All rights reserved - 554. You may not reproduce, distribute or exploit the contents in any form without written permission by copyright owner. Copyright infringers may face civil and criminal liability TEST - 20 (REVISION CUM Total Marks 200 MOCK TEST - 5) :

72 Key Biodiversity Areas (KBAs) are nationally identified sites of global significance. They include 1. Alliance for Zero Extinction (AZE) sites 2. Important Plant Areas (IPAs) 3. Important Sites for Freshwater Biodiversity (ISFB) 4. Important Bird and Biodiversity Areas (IBAs)

Select the correct answer using the codes below. A. 1 and 3 only B. 2, 3 and 4 only C. 1 and 4 only D. 1, 2, 3 and 4

Correct Answer : D

Answer Justification :

Justification: The identification of KBAs is an important approach to address biodiversity conservation at the site scale i.e. at the level of individual protected areas, concessions and land management units.

KBAs are identified using globally standardised criteria and thresholds, and have clearly defined boundaries. There is no maximum or minimum size of sites, because appropriate size varies according to the socio-economic criteria, such as land use and tenure.

KBAs are seen as an ‘umbrella’ designation, which includes globally important sites for different taxa and realms as described in the Q Statement.

NAVEEN KUMAR N S - [email protected] - 9632470826

Q Source: http://biodiversitya-z.org/content/key-biodiversity-areas-kba

offline.insightsias.com 81 © Insights Active Learning | All rights reserved - 554. You may not reproduce, distribute or exploit the contents in any form without written permission by copyright owner. Copyright infringers may face civil and criminal liability TEST - 20 (REVISION CUM Total Marks 200 MOCK TEST - 5) :

73 Consider the following statements about Neutrinos. 1. They are used to trigger fission in nuclear reactors. 2. Human body cannot tolerate exposure to Neutrinos. 3. They are completely massless.

Choose the correct answer using the codes below. A. 1 and 2 only B. 2 and 3 only C. 1 and 3 only D. None of the above

Correct Answer : D

Answer Justification :

Justification: Neutrinos are the information bearers of the universe — which are almost never lost in their path. India’s effort in studying neutrinos at INO may help us unravel the deepest mystery of the universe — why there is more matter than antimatter in the universe.

Neutrinos are the least harmful of all elementary particles, as they almost never react with solid bodies.

Also, people tend to confuse the “neutrino” for the “neutron”. This has also led to the confusion that neutrinos can be weaponised, which is far from the truth. They are not used to trigger fission reactions, but are instead produced as a result of the reactions.

Neutrinos have mass and as shown recently, they change their masses too.

Neutrinos, named as such because they are electrically neutral, are leptons, and so are not affected by the strong force either. NAVEEN KUMAR N S - [email protected] - 9632470826

Q Source: Frequently in news

offline.insightsias.com 82 © Insights Active Learning | All rights reserved - 554. You may not reproduce, distribute or exploit the contents in any form without written permission by copyright owner. Copyright infringers may face civil and criminal liability TEST - 20 (REVISION CUM Total Marks 200 MOCK TEST - 5) :

74 Aichi Biodiversity Targets are a set of 20 global targets under the

A. Strategic Plan for Biodiversity 2011-2020 B. Sustainable Development Goals (SDGs) C. UNFCCC Climate Commitments D. UNEP Plan for Global Action

Correct Answer : A

Answer Justification :

Learning: They are grouped under five strategic goals:

Address the underlying causes of biodiversity loss by mainstreaming biodiversity across government and society.

Reduce the direct pressures on biodiversity and promote sustainable use.

Improve the status of biodiversity by safeguarding ecosystems, species and genetic diversity.

Enhance the benefits to all from biodiversity and ecosystem services.

Enhance implementation through participatory planning, knowledge management and capacity building.

The above information comes from the Secretariat of the Convention on Biological Diversity (CBD Secretariat) (2014).

NAVEENQ Source: http://www.biodiversitya-z.org/content/aichi-biodiversity-targetsKUMAR N S - [email protected] - 9632470826

offline.insightsias.com 83 © Insights Active Learning | All rights reserved - 554. You may not reproduce, distribute or exploit the contents in any form without written permission by copyright owner. Copyright infringers may face civil and criminal liability TEST - 20 (REVISION CUM Total Marks 200 MOCK TEST - 5) :

75 Consider the following about the Bharhut Stupa. 1. The Shungas improvised the art work at Bharhut. 2. Jataka tales have been depicted on its railings. 3. Queen Maya’s dream has been depicted at Bharhut.

Select the correct answer using the codes below. A. 1 and 2 only B. 2 and 3 only C. 1 and 3 only D. 1, 2 and 3

Correct Answer : D

Answer Justification :

Justification: Statement 1: The Bharhut stupa (in MP) may have been first built by the Maurya king Ashoka in the 3rd century BCE, but many works of art were apparently added during the Shunga period, with many friezes from the 2nd century BCE.

Statement 2: They contain numerous birth stories of the Buddha's previous lives, or Jataka tales. Many of them are in the shape of large, round medallions.

Statement 3: Queen Maya’s dream, preceding the birth of the Buddha, is also a major theme on the railing of the Bharhut “stupa”.

NAVEEN KUMAR N S - [email protected] - 9632470826

Q Source: Chapter 4: 11th NCERT: An Introduction to Indian Arts

offline.insightsias.com 84 © Insights Active Learning | All rights reserved - 554. You may not reproduce, distribute or exploit the contents in any form without written permission by copyright owner. Copyright infringers may face civil and criminal liability TEST - 20 (REVISION CUM Total Marks 200 MOCK TEST - 5) :

76 Dhammacakkappavattana Sutta is

A. A set of secular codes for Buddhist monks B. Considered to be a record of the first teaching given by Gautama Buddha after he attained enlightenment. C. The compilation of philosophical work on Buddhist teachings by religious scholars D. The last teaching given by the Buddha before his Mahaparinirvana

Correct Answer : B

Answer Justification :

Learning: The Setting in Motion of the Dharmachakra Sutra or Promulgation of the Law Sutra) is a Buddhist text that is considered to be a record of the first teaching given by Gautama Buddha after he attained enlightenment.

According to tradition, the Buddha gave this teaching in Sarnath, India, to the "five ascetics", his former companions with whom he had spent six years practicing austerities.

The main topic of this sutra is the Four Noble Truths, which are the central teachings of Buddhism that provide a unifying theme, or conceptual framework, for all of Buddhist thought. This sutra also introduces the Buddhist concepts of the Middle Way, impermanence, and dependent origination.

Q Source: 11th NCERT: An Introduction to Indian Arts

NAVEEN KUMAR N S - [email protected] - 9632470826

offline.insightsias.com 85 © Insights Active Learning | All rights reserved - 554. You may not reproduce, distribute or exploit the contents in any form without written permission by copyright owner. Copyright infringers may face civil and criminal liability TEST - 20 (REVISION CUM Total Marks 200 MOCK TEST - 5) :

77 The ex-officio Chairman of the Civil Services Board is

A. Prime Minister B. Minister of Personnel and Grievances C. Cabinet Secretary D. Director, LBSNAA

Correct Answer : C

Answer Justification :

Justification & Learning: Cabinet Secretary is also the head of the civil services. The Secretaries felt it necessary to keep the Cabinet Secretary informed of developments from time to time.

Committees are constituted for discussing specific matters and proposals emanating from various Secretaries to the Government and meetings are held under the chairmanship of the Cabinet Secretary.

Among the inter-Ministerial matters, the coordination is required for:

Removing difficulties.

Removing differences.

Overcoming delays.

Coordination in administrative action.

Coordination of policies. NAVEEN KUMAR N S - [email protected] - 9632470826 These are achieved with the help of the Cabinet Secretary.

Q Source: Additional Research: Indian Polity: M Laxmikanth

offline.insightsias.com 86 © Insights Active Learning | All rights reserved - 554. You may not reproduce, distribute or exploit the contents in any form without written permission by copyright owner. Copyright infringers may face civil and criminal liability TEST - 20 (REVISION CUM Total Marks 200 MOCK TEST - 5) :

78 Which of the following is/are landlocked countries with a monarchy? 1. Bolivia 2. Kyrgyzstan 3. Afghanistan 4. Zimbabwe

Select the correct answer using the codes below. A. 2 and 4 only B. 1 and 3 only C. 2 only D. None of the above

Correct Answer : D

Answer Justification :

Justification: All countries are landlocked countries.

Zimbabwe is a Dominant-party presidential republic.

Kyrgyzstan is a unitary parliamentary republic.

Bolivia is a unitary presidential constitutional republic. Afghanistan is a unitary presidential Islamic republic.

Q Source: Chapter 5: 12th NCERT: Contemporary World Politics

NAVEEN KUMAR N S - [email protected] - 9632470826

offline.insightsias.com 87 © Insights Active Learning | All rights reserved - 554. You may not reproduce, distribute or exploit the contents in any form without written permission by copyright owner. Copyright infringers may face civil and criminal liability TEST - 20 (REVISION CUM Total Marks 200 MOCK TEST - 5) :

79 Which of the following features of the Indian constitution are borrowed from the USA Constitution? 1. The Idea of a federal polity 2. Written Constitution 3. The Idea of Rule of Law 4. Fundamental Rights

Select the correct answer using the codes below: A. 1, 2 and 4 only B. 2 and 3 only C. 3 and 4 only D. None of the above

Correct Answer : B

Answer Justification :

Justification: The Idea of a federal polity was borrowed from Canada. So, Statement 1 is incorrect.

The Idea of rule of law was borrowed from England. So, 3 is wrong.

Learning: From U.K.

Nominal Head – President (like Queen)

Cabinet System of Ministers

Post of PM

Parliamentary Type of Govt.

NAVEEN KUMAR N S - [email protected] - 9632470826 Bicameral Parliament

Lower House more powerful

Council of Ministers responsible to Lowe House

Speaker in Lok Sabha

From U.S.A.

Written Constitution offline.insightsias.com 88 Executive head of state known as President and his being the Supreme Commander of the © Insights Active Learning | All rights reserved - 554. You may not reproduce, distribute or exploit the contents in any form without written permission by copyright owner. Copyright infringers may face civil and criminal liability TEST - 20 (REVISION CUM Total Marks 200 MOCK TEST - 5) :

Armed Forces

Vice- President as the ex-officio Chairman of Rajya Sabha

Fundamental Rights

Supreme Court

Provision of States

Independence of Judiciary and judicial review

Preamble

Removal of Supreme court and High court Judges

Q Source: 11th NCERT: Indian Constitution

NAVEEN KUMAR N S - [email protected] - 9632470826

offline.insightsias.com 89 © Insights Active Learning | All rights reserved - 554. You may not reproduce, distribute or exploit the contents in any form without written permission by copyright owner. Copyright infringers may face civil and criminal liability TEST - 20 (REVISION CUM Total Marks 200 MOCK TEST - 5) :

80 What is an Eco-sensitive Zone (ESZ) as notified by the Ministry of Environment and Forests (MoEF) frequently?

A. It is the area around a national park or sanctuary where developmental activities are regulated B. It is a zone of high endemism of species C. It is the “Core zone” inside a protected area where no activities are allowed D. It is an ecological hotspot which is internationally recognized as a part of protected areas network

Correct Answer : A

Answer Justification :

Learning: As per MOEF 2011 guidelines, it is the area around a national park or sanctuary upto 10 kms where developmental activities are regulated (not prohibited) to:

Minimize impact of ecologically harmful developmental activities (e.g. mining) on fragile ecosystem encompassing the area

Protects biodiversity by avoiding fragmentation of habitat

Act as a transition zone between areas of high protection (inside) and low protection outside

Act as a "shock absorber" for the protected areas

Q Source: http://envfor.nic.in/content/esz-notifications

NAVEEN KUMAR N S - [email protected] - 9632470826

offline.insightsias.com 90 © Insights Active Learning | All rights reserved - 554. You may not reproduce, distribute or exploit the contents in any form without written permission by copyright owner. Copyright infringers may face civil and criminal liability TEST - 20 (REVISION CUM Total Marks 200 MOCK TEST - 5) :

81 The Directive Principles of State Policy (DPSP) provide for 1. Fundamental principles of governance 2. Constitutional rights and legal entitlements of marginalized sections of society

Which of the above is/are correct? A. 1 only B. 2 only C. Both 1 and 2 D. None

Correct Answer : A

Answer Justification :

Justification: The constitution itself mentions that Directive Principles of State Policy (DPSP) are fundamental to the governance of India, as they provides for the essential principles like equity, fairness, welfare orientation etc. So, 1 is correct.

2 will be incorrect because DPSP only gives guidance to the state in the formation of policies. The rights and entitlements can only be provided for in a law enacted by the legislature which gives force to the provisions of DPSP.

Q Source: Revision: Indian Polity: M Laxmikanth

NAVEEN KUMAR N S - [email protected] - 9632470826

offline.insightsias.com 91 © Insights Active Learning | All rights reserved - 554. You may not reproduce, distribute or exploit the contents in any form without written permission by copyright owner. Copyright infringers may face civil and criminal liability TEST - 20 (REVISION CUM Total Marks 200 MOCK TEST - 5) :

82 A country is likely to have multiple time zones if

A. It has a large area. B. It is closer to the equator. C. If it is closer to the poles. D. None of the above has any bearing on the time zones of a country.

Correct Answer : A

Answer Justification :

Justification: If a nation is spread on a large geographical area latitudinally, the Sun will be at different positions in the sky for different latitudes on the nation at the same time. So, there is a possibility of having multiple time zones. For e.g. in Russia, there are about 11 time zones.

The same cannot be said for longitude as it does not concern with the movement of the Sun or daylight.

Q Source: Revision: 11th NCERT: India Physical Geography

NAVEEN KUMAR N S - [email protected] - 9632470826

offline.insightsias.com 92 © Insights Active Learning | All rights reserved - 554. You may not reproduce, distribute or exploit the contents in any form without written permission by copyright owner. Copyright infringers may face civil and criminal liability TEST - 20 (REVISION CUM Total Marks 200 MOCK TEST - 5) :

83 Which of these taxation systems is most closely related to what India follows?

A. Proportional system B. Regressive system C. Progressive system D. Neither progressive nor proportional

Correct Answer : C

Answer Justification :

Learning: Progressive taxation method has increasing rates of tax for increasing value or volume on which the tax is being imposed.

Indian income tax is a typical example of it. The idea here is less tax on the people who earn less and higher tax on the people who earn more—classifying income earners into different slabs.

This method is believed o discourage more earnings by the individual to support low growth and development unintentionally. Being poor is rewarded while richness is punished.

Tax payers also start evading tax by showing lower unreal income. But from different angles this tax is pro-poor and taxes people according to their affordability/sustainability. This is the most popular taxation method in the world and a populist one, too.

Regressive system is just the reverse of this.

Proportional Taxation - In such a taxation method, there is neither progression nor regression from the point of view rate of taxes point of view. Such taxes have fixed rates for every level of income or production, they are neutral from the poor or rich point view or from the point of view of the levels of production.

Q Source: Chapter 17: Indian Economy: Ramesh Singh

NAVEEN KUMAR N S - [email protected] - 9632470826

offline.insightsias.com 93 © Insights Active Learning | All rights reserved - 554. You may not reproduce, distribute or exploit the contents in any form without written permission by copyright owner. Copyright infringers may face civil and criminal liability TEST - 20 (REVISION CUM Total Marks 200 MOCK TEST - 5) :

84 The writ of “Quo Warranto” can be issued to address which of the following?

A. If someone has been detailed illegally B. If a legislator holds such office which makes him ineligible to become an MP C. If an official is not doing her duties properly D. If a lower court has usurped illegal jurisdiction

Correct Answer : B

Answer Justification :

Learning: Quo warranto means “by what warrant”? This means that Supreme Court and High Court may issue the writ which restrains the person or authority to act in an office which he / she is not entitled to.

This writ is applicable to the public offices only.

Option C is mandamus, and Option D is certiorari.

Q Source: Chapter 7: Indian Polity: Laxmikanth

NAVEEN KUMAR N S - [email protected] - 9632470826

offline.insightsias.com 94 © Insights Active Learning | All rights reserved - 554. You may not reproduce, distribute or exploit the contents in any form without written permission by copyright owner. Copyright infringers may face civil and criminal liability TEST - 20 (REVISION CUM Total Marks 200 MOCK TEST - 5) :

85 The Hindu Kush Himalayan Monitoring and Assessment Programme (HIMAP) is being initiated and coordinated by

A. United Nations Environment Programme (UNEP) B. World Economic Forum and IUCN Jointly C. Global Conservation Fund - Conservation International D. International Centre for Integrated Mountain Development (ICIMOD)

Correct Answer : D

Answer Justification :

Learning: HIMAP is aimed towards producing a Comprehensive Assessment of the Hindu Kush Himalayan (HKH) Region.

This assessment will assess the current state of knowledge of the HKH region, address critical data gaps and increase the understanding of various drivers of change and their impacts.

Based on the assessment, HIMAP will recommend a set of practically oriented policy recommendations.

The assessment process will involve a broad and diverse group of researchers, practitioners and policy makers, and culminate in a major publication with a highly reputed international publishing house in 2017.

Q Source: http://www.hi-map.org/

NAVEEN KUMAR N S - [email protected] - 9632470826

offline.insightsias.com 95 © Insights Active Learning | All rights reserved - 554. You may not reproduce, distribute or exploit the contents in any form without written permission by copyright owner. Copyright infringers may face civil and criminal liability TEST - 20 (REVISION CUM Total Marks 200 MOCK TEST - 5) :

86 Delimitation of constituencies and allocation of seats in the Parliament and state legislatures are dealt by

A. Representation of Peoples Act 1950 (RPA Act 1950) B. Only the constitution of India C. Rules Made by the Presiding officer of Each house D. Orders issued by the Secretary-General of the corresponding houses

Correct Answer : A

Answer Justification :

Learning: The RPA provides for the following:

Qualification of voters.

Preparation of electoral rolls.

Delimitation of constituencies.

Allocation of seats in the Parliament and state legislatures.

RPA Act 1951 provides for

Actual conduct of elections.

Administrative machinery for conducting elections. NAVEEN KUMAR N S - [email protected] - 9632470826 Poll.

Election offences.

Election disputes.

By-elections.

Registration of political parties.

Q Source: Additional Research: 10th NCERT Democratic Politics offline.insightsias.com 96 © Insights Active Learning | All rights reserved - 554. You may not reproduce, distribute or exploit the contents in any form without written permission by copyright owner. Copyright infringers may face civil and criminal liability TEST - 20 (REVISION CUM Total Marks 200 MOCK TEST - 5) :

87 Often seen in news, Minimum Alternate Tax (MAT) is

A. A form of direct tax B. Imposed on High Net Worth Individuals (HNIs) C. Applied on goods for import and export in foreign trade D. All of the above

Correct Answer : A

Answer Justification :

Learning: Income tax is paid as per the provisions of the Income Tax Act (IT Act) but companies calculate their profit (through profit and loss account) as per the provisions of the Companies Act.

The IT Act allows several kinds of exemptions and other incentives from total income together with deductions on the gross income.

As a result of these exemptions, deductions and other incentives under IT Act, companies show their taxable income either ‘nil’ or ‘negative’, and this way, the ‘zero tax’ companies emerge and pay no taxes even though they may be paying huge dividends (under the Companies Act) to their shareholders.

To bring such companies under the income tax, MAT was imposed accordingly.

It is applicable on all companies except those engaged in infrastructure and power sectors, free trade zones, charitable activities, venture and angel funds. Foreign companies with income sources in India also come under it.

Q Source: Chapter 17: Indian Economy: Ramesh Singh

NAVEEN KUMAR N S - [email protected] - 9632470826

offline.insightsias.com 97 © Insights Active Learning | All rights reserved - 554. You may not reproduce, distribute or exploit the contents in any form without written permission by copyright owner. Copyright infringers may face civil and criminal liability TEST - 20 (REVISION CUM Total Marks 200 MOCK TEST - 5) :

88 Consider the following about the spread of Jainism. 1. Mahavira allowed both men and women to join Jain Sanghas. 2. The then rulers of Southern Indian patronized Jainism. 3. Chandragupta Maurya is said to have persecuted Jainas on account of their severe ascetic practices.

Select the correct answer using the codes below. A. 1 and 2 only B. 1 and 3 only C. 2 only D. None of the above

Correct Answer : A

Answer Justification :

Justification: Mahavira organised the Sangha to spread his teachings. He admitted both men and women in the Sangha, which consisted of both monks and lay followers.

The rapid spread of Jainism was due to the dedicated work of the members of the Sangha. It spread rapidly in Western India and Karnataka.

Statement 3: Chandragupta Maurya, Kharavela of Kalinga and the royal dynasties of south India such as the Gangas, the Kadambas, the Chalukyas and the Rashtrakutas patronized Jainism.

By the end of the fourth century B.C., there was a serious famine in the Ganges valley. Many Jain monks led by Bhadrabagu and Chandragupta Maurya came to Sravana Belgola in Karnataka.

Those who stayed back in north India were led by a monk named Sthulabahu who changed the code of conduct for the monks. This led to the division of Jainism into two Svetambaras and Digambaras.

Q Source: Revision: Chapter 4: 11th Std. TamilNadu History Textbook

NAVEEN KUMAR N S - [email protected] - 9632470826

offline.insightsias.com 98 © Insights Active Learning | All rights reserved - 554. You may not reproduce, distribute or exploit the contents in any form without written permission by copyright owner. Copyright infringers may face civil and criminal liability TEST - 20 (REVISION CUM Total Marks 200 MOCK TEST - 5) :

89 Which of these nations lie both in Northern and Southern Hemisphere? 1. Brazil 2. Maldives 3. Thailand 4. Morocco

Select the correct answer using the codes below. A. 1 and 2 only B. 3 and 4 only C. 1, 2 and 3 only D. 2, 3 and 4 only

Correct Answer : A

Answer Justification :

Learning:

NAVEEN KUMAR N S - [email protected] - 9632470826

The image shows equator passing through countries, these are the ones that lie in both hemispheres.

offline.insightsias.com 99 © Insights Active Learning | All rights reserved - 554. You may not reproduce, distribute or exploit the contents in any form without written permission by copyright owner. Copyright infringers may face civil and criminal liability TEST - 20 (REVISION CUM Total Marks 200 MOCK TEST - 5) :

90 The major tectonic plates that converge on Western Asia are 1. Indian Plate 2. African plate 3. Arabian Plate 4. Pacific plate

Select the correct answer using the codes below. A. 2 and 3 only B. 3 and 4 only C. 1, 2 and 3 only D. 2, 3 and 4 only

Correct Answer : A

Answer Justification :

Background: The outer shell of the earth, the lithosphere, is broken up into tectonic plates. The seven major plates are the African plate, Antarctic plate, Eurasian plate, Indo-Australian plate, North American plate, Pacific plate and South American plate.

Justification: The West Asian convergence includes the African, Eurasian, and Arabian plates. The boundaries between the tectonic plates make up the Azores-Gibraltar Ridge, extending across North Africa, the Red Sea, and into Iran.

The Arabian Plate is moving northward into the Anatolian plate (Turkey) at the East Anatolian Fault, and the boundary between the Aegean and Anatolian plate in eastern Turkey is also seismically active.

Q Source: Map-based: West Asia

NAVEEN KUMAR N S - [email protected] - 9632470826

offline.insightsias.com 100 © Insights Active Learning | All rights reserved - 554. You may not reproduce, distribute or exploit the contents in any form without written permission by copyright owner. Copyright infringers may face civil and criminal liability TEST - 20 (REVISION CUM Total Marks 200 MOCK TEST - 5) :

91 The United Nations High-level Political Forum on Sustainable Development (HLPF) was established as a result of the mandate in

A. Rio+ 20 Conference, 2012 B. UN Conference on the Human Environment (Stockholm 1972) C. Convention on Biological Diversity (1992) D. Paris Conference, 2014

Correct Answer : A

Answer Justification :

Learning: In order to ensure compliance to the agreements at Rio (particularly the Rio Declaration on Environment and Development and Agenda 21), delegates to the Earth Summit established the Commission on Sustainable Development (CSD).

In 2013, the CSD was replaced by the High-level Political Forum on Sustainable Development that meets every year as part of the ECOSOC meetings, and every fourth year as part of the General Assembly meetings.

The Forum meets annually under the auspices of the Economic and Social Council for eight days, including a three-day ministerial segment and every four years at the level of Heads of State and Government under the auspices of the General Assembly for two days.

The HLPF is the main United Nations platform on sustainable development and it has a central role in the follow-up and review of the 2030 Agenda for Sustainable Development the Sustainable Development Goals (SDGs) at the global level.

Q Source: https://sustainabledevelopment.un.org/hlpf

https://en.wikipedia.org/wiki/Earth_Summit

NAVEEN KUMAR N S - [email protected] - 9632470826

offline.insightsias.com 101 © Insights Active Learning | All rights reserved - 554. You may not reproduce, distribute or exploit the contents in any form without written permission by copyright owner. Copyright infringers may face civil and criminal liability TEST - 20 (REVISION CUM Total Marks 200 MOCK TEST - 5) :

92 Special safeguard mechanism (SSM) as under WTO relates to

A. Governments recognizing other countries’ measures as acceptable even if they are different from their own, so long as an equivalent level of protection is provided B. Support for farmers that is not linked to prices or production C. Licensing companies or individuals other than the patent owner to use the rights of the patent D. Allowing developing countries to raise tariffs temporarily to deal with import surges or price falls

Correct Answer : D

Answer Justification :

Learning: WTO’s Special Safeguard Mechanism (SSM) is a protection measure allowed for developing countries to take contingency restrictions against agricultural imports that are causing injuries to domestic farmers.

The contingency measure is imposition of tariff if the import surge causes welfare loss to the domestic poor farmers. The design and use of the SSM is an area of conflict under the WTO.

At the Doha Ministerial Conference, the developing countries were given a concession to adopt a Special Safeguard Mechanism (SSM) besides the existing safeguards (like the Special Agricultural Safeguard or the SSG). This SSM constituted an important part of the promises offered to the developing world at Doha (known as Doha Development Agenda) and the Doha MC became known as a development round.

The SSG was available to all countries- both developing and developed whereas the SSM is allowable only to the developing countries.

Q Source: Revision: Indian Economy: Ramesh Singh

NAVEEN KUMAR N S - [email protected] - 9632470826

offline.insightsias.com 102 © Insights Active Learning | All rights reserved - 554. You may not reproduce, distribute or exploit the contents in any form without written permission by copyright owner. Copyright infringers may face civil and criminal liability TEST - 20 (REVISION CUM Total Marks 200 MOCK TEST - 5) :

93 ICESCAPE Mission, is a

A. Multi-year NASA shipborne project B. Himalayan mineral excavation programme C. Extension of the Maitri mission of India in Arctic D. Joint expedition of DRDO and ISRO into understanding satellite launched from remote and cold locations

Correct Answer : A

Answer Justification :

Learning: Impacts of Climate on the Eco-Systems and Chemistry of the Arctic Pacific Environment (ICESCAPE) is a multi-year NASA shipborne project.

The central science question of this program is, “What is the impact of climate change (natural and anthropogenic) on the biogeochemistry and ecology of the Chukchi and Beaufort seas?” While both of these regions are experiencing significant changes in the ice cover, their biogeochemical response will likely be quite different due to their distinct physical, chemical, and biological differences.

ICESCAPE will pursue the above central science question and associated issues through an interdisciplinary, cross cutting approach integrating field expeditions, modeling, and satellite remote sensing.

Q Source: https://www.nasa.gov/multimedia/imagegallery/image_feature_1921.html

https://espo.nasa.gov/icescape/content/ICESCAPE_0

NAVEEN KUMAR N S - [email protected] - 9632470826

offline.insightsias.com 103 © Insights Active Learning | All rights reserved - 554. You may not reproduce, distribute or exploit the contents in any form without written permission by copyright owner. Copyright infringers may face civil and criminal liability TEST - 20 (REVISION CUM Total Marks 200 MOCK TEST - 5) :

94 Our Planet, Atlas and Global Environment Outlook (GEO) are flagship publications of

A. United Nations Environment Programme (UNEP) B. International Union for Conservation of Nature (IUCN) C. Greenpeace D. World Wildlife Fund (WWF)

Correct Answer : A

Answer Justification :

Learning: Our Planet is UNEP's magazine for environmentally sustainable development. Each issue of Our Planet concentrates on one specific theme, tying into international conferences (for e.g. the 10th anniversary of the Basel Convention), meetings (e.g. the Commission on Sustainable Development), and events.

The Atlas of Our Changing Environment publications use a combination of ground photographs, current and historical satellite images, and narrative based on extensive scientific evidence to illustrate how humans have altered their surroundings.

Global Environment Outlook (GEO) is a series of reports on the environment issued periodically by the United Nations Environmental Programme (UNEP). The GEO project was initiated in response to the environmental reporting requirements of UN Agenda 21.

Q Source: http://web.unep.org/geo/

NAVEEN KUMAR N S - [email protected] - 9632470826

offline.insightsias.com 104 © Insights Active Learning | All rights reserved - 554. You may not reproduce, distribute or exploit the contents in any form without written permission by copyright owner. Copyright infringers may face civil and criminal liability TEST - 20 (REVISION CUM Total Marks 200 MOCK TEST - 5) :

95 Consider the following statements. 1. Asoka’s Dhamma contained the ideal of a ‘welfare state’. 2. Ashoka propagated Dhamma with the help of officials known as Dhamma Mahamattas. 3. Prohibition of animal sacrifices was a part of Asoka’s Dhamma.

Select the correct answer using the codes below. A. 1 and 2 only B. 2 and 3 only C. 1 and 3 only D. 1, 2 and 3 only

Correct Answer : D

Answer Justification :

Justification: Statement 2: Efficient organization of administration in the direction of social welfare and maintenance of constant contact with people through the system of Dhammayatras was an important feature of Ashoka’s dhamma.

The Dhamma Mahamatras were asked by Asoka to take steps against unjust imprisonment. Remission of sentences is also mentioned in Asoka’s inscriptions. But, he did not advocate non- punishment. So, 3 is wrong.

Other features of Ashoka’s dhamma were:

Service to father and mother, practice of ahimsa, love of truth, reverence to teachers and good treatment of relatives.

Prohibition of animal sacrifices and festive gatherings and avoiding expensive and meaningless ceremonies and rituals.

NAVEENHumane KUMAR treatment N of S servants - [email protected] by masters and prisoners by government - 9632470826 officials.

Consideration and non-violence to animals and courtesy to relations and liberality to Brahmins.

Q Source: Revision: Chapter 6: 11th Std. TamilNadu History Textbook

offline.insightsias.com 105 © Insights Active Learning | All rights reserved - 554. You may not reproduce, distribute or exploit the contents in any form without written permission by copyright owner. Copyright infringers may face civil and criminal liability TEST - 20 (REVISION CUM Total Marks 200 MOCK TEST - 5) :

96 CNX Nifty is used extensively by investors in India and around the world as a barometer of the Indian capital markets. It is prepared by

A. Reserve Bank of India (RBI) B. Securities and Exchange Board of India (SEBI) C. National Stock Exchange of India D. Department of Financial Services, Union Government

Correct Answer : C

Answer Justification :

Learning: The NIFTY 50 covers 22 sectors of the Indian economy and offers investment managers exposure to the Indian market in one portfolio.

The base period for the CNX Nifty index is 1995.

NSE was the first exchange in the country to provide a modern, fully automated screen-based electronic trading system which offered easy trading facility to the investors spread across the length and breadth of the country.

NSE was set up by a group of leading Indian financial institutions at the behest of the government of India to bring transparency to the Indian capital market.

Q Source: Frequently in news

NAVEEN KUMAR N S - [email protected] - 9632470826

offline.insightsias.com 106 © Insights Active Learning | All rights reserved - 554. You may not reproduce, distribute or exploit the contents in any form without written permission by copyright owner. Copyright infringers may face civil and criminal liability TEST - 20 (REVISION CUM Total Marks 200 MOCK TEST - 5) :

97 Consider the following statements. 1. In both urban and rural India, there is a greater proportion of male workers than female workers. 2. Majority of female workers are from rural areas.

Which of the above is/are correct? A. 1 only B. 2 only C. Both 1 and 2 D. None

Correct Answer : C

Answer Justification :

Justification: Of the total 402 million workers, 275 million are males and 127 million females.

This would mean that 51.7 percent of the total males and 25.6 percent of the total females are workers. The number of female workers is about less than half the number of male workers. In terms of proportion, 68.4 percent of the workers are males and 31.6 percent females.

Main workers constitute 77.8 percent of the total workers. The remaining are marginal workers. Among the main workers, female workers, are only 23.3 % and 76.7% are male workers.

Majority of female workers (87.3 percent) are from rural areas. This is also twice that of male workers, which may be due to their being employed predominantly in activities like cultivation and agricultural labour.

In the urban areas, majority of female workers are engaged in Households industry and other work. NAVEEN KUMAR N S - [email protected] - 9632470826

Interestingly, among marginal workers females outnumber the males. In three of the four categories, viz. cultivators, agricultural labourers and household industries, female marginal workers outnumber male workers.

Q Source: Additional Research: Indian Economy: Ramesh Singh

offline.insightsias.com 107 © Insights Active Learning | All rights reserved - 554. You may not reproduce, distribute or exploit the contents in any form without written permission by copyright owner. Copyright infringers may face civil and criminal liability TEST - 20 (REVISION CUM Total Marks 200 MOCK TEST - 5) :

98 The Aranyaka texts are so-called because

A. They cannot be read by those other than Siddhas. B. They contain metaphysics C. They are works to be read in the forests. D. They appear after Upanishads

Correct Answer : C

Answer Justification :

Learning: Aranyakas are generally the concluding portions of the several Brahmanas, but on account of their distinct character, contents and language deserve to be reckoned as a distinct category of literature.

They are partly included in the Brahmanas themselves, but partly they are recognized as independent works. Aranyaka literature is rather small as compared to the Brahmanas.

The term Aranyaka is derived from the word 'Aranya' meaning 'forest'. The Aranyaka texts are so-called because 'they were works to be read in the forest' in contradistinction to the regular Brahmanas, which were to be read in the village.

Whereas the Brahmanas deal with the huge bulk of sacrificial paraphernalia which represents Karma-Kanda, the Aranyakas and Upanishads, on the other hand, chiefly deal with the philosophical and theosophical speculations which represent Jnana-Kanda.

Q Source: Revision: NIOS Culture material

NAVEEN KUMAR N S - [email protected] - 9632470826

offline.insightsias.com 108 © Insights Active Learning | All rights reserved - 554. You may not reproduce, distribute or exploit the contents in any form without written permission by copyright owner. Copyright infringers may face civil and criminal liability TEST - 20 (REVISION CUM Total Marks 200 MOCK TEST - 5) :

99 Consider the following funds under UNFCCC and their parent management fund. 1. Special Climate Change Fund (SCCF): Green Climate Fund (GCF) 2. Adaptation Fund (AF): Green Environment Facility (GEF) 3. Least Developed Countries Fund (LDCF): Kyoto Protocol

Select the correct answer using the codes below. A. 1 only B. 2 and 3 only C. 1 and 2 only D. 1 and 3 only

Correct Answer : A

Answer Justification :

Background: The UNFCCC, under its Article 11, states that the operation of the Financial Mechanism is entrusted to one or more existing international entities.

The operation of the Financial Mechanism is partly entrusted to the Global Environment Facility (GEF).

At COP 17 Parties decided to designate the Green Climate Fund (GCF) as an operating entity of the Financial Mechanism of the Convention, in accordance with Article 11 of the Convention.

The Financial Mechanism is accountable to the COP, which decides on its climate change policies, programme priorities and eligibility criteria for funding.

The Kyoto Protocol also recognizes, under its Article 11, the need for the Financial Mechanism to fund activities by developing country Parties.

Justification: In addition to providing guidance to the GEF, Parties have established four special funds: the Special Climate Change Fund (SCCF), the Least Developed Countries Fund (LDCF), both managed by the GEF, and the GCF under the Convention; and the Adaptation Fund (AF) under the NAVEENKyoto Protocol. KUMAR N S - [email protected] - 9632470826

The Adaptation Fund (AF) was established in 2001 to finance concrete adaptation projects and programmes in developing country Parties to the Kyoto Protocol that are particularly vulnerable to the adverse effects of climate change.

The Adaptation Fund is financed with a share of proceeds from the clean development mechanism (CDM) project activities and other sources of funding. The share of proceeds amounts to 2 per cent of certified emission reductions (CERs) issued for a CDM project activity.

Q Source http://unfccc.int/cooperation_and_support/financial_mechanism/items/2807.php

offline.insightsias.com 109 © Insights Active Learning | All rights reserved - 554. You may not reproduce, distribute or exploit the contents in any form without written permission by copyright owner. Copyright infringers may face civil and criminal liability TEST - 20 (REVISION CUM Total Marks 200 MOCK TEST - 5) :

100 The northern India of 6th century B.C. consisted of a large number of independent kingdoms, some of which were republics which consisted of only one tribe. Which of the following was NOT one of them?

A. B. Licchavis C. Mallas D. Vakatakas

Correct Answer : D

Answer Justification :

Learning: Some of the republics consisted of only one tribe like the Sakyas, Licchavis and Mallas. In these republics the power of decision vested with the Public Assembly which was composed of the tribal representatives or head of families

All decisions were by a majority vote.

Apart from republics, there were also monarchical forms of government. The Buddhist literature Anguttara Nikaya gives a list of sixteen great kingdoms called ‘Sixteen Mahajanapadas’.

In course of time, the small and weak kingdoms either submitted to the stronger rulers or gradually got eliminated. Finally in the mid 6th century B.C., only four kingdoms – Vatsa, Avanti, Kosala and Magadha survived.

Q Source: Revision: Chapter 5: 11th Std. TamilNadu History Textbook

NAVEEN KUMAR N S - [email protected] - 9632470826

offline.insightsias.com 110 © Insights Active Learning | All rights reserved - 554. You may not reproduce, distribute or exploit the contents in any form without written permission by copyright owner. Copyright infringers may face civil and criminal liability TEST - 20 (REVISION CUM Total Marks 200 MOCK TEST - 5) :

NAVEEN KUMAR N S - [email protected] - 9632470826

offline.insightsias.com 111 © Insights Active Learning | All rights reserved - 554. You may not reproduce, distribute or exploit the contents in any form without written permission by copyright owner. Copyright infringers may face civil and criminal liability